You are on page 1of 96

MATHEMATICS

Serial Page
No.
PART - V No.

1. Differential Equation 1

2. Probability 47

CLASS - XII
DIFFERENTIAL EQUATIONS 1
UNIT

 Theory

 Drill Exercises

 Exercises

 Single Correct Type Questions

 Multiple Correct Type Questions

 Matrix Match Type Questions.

 Comprehension Type Questions

 Integer Type Questions

 Previous Years Questions

 Answer Key
DIFFERENTIAL EQUATION

Definitions
An equation that involves independent and dependent variables and the
derivatives of the dependent variables is called a differential equation.
Ordinary and Partial Differential equation : A differential equation is said to be
ordinary, if the differential coefficients have reference to a single independent
variable only and it is said to be partial if there are two or more independent
variables .

d2 y dy
eg. 2
3  2y  0 is an ordinary diffrential equation
dx dx

y x y 2 z 2 z
  0 ; 2
 2
 x2  y are partial differential equation .
x y z x y
Order and Degree of Diffrential Equation
Order : The order of a differential equation is the order of the highest
differential coefficient occuring in it.
Degree : which is expressed or can be expressed as a polynomial in the derivatives
is the degree of the highest order derivative occuring in it, after it has been
expressed in a form free from radicals & fractions so far as derivatives are
concerned, thus the differential equation :

6 1/4
d2 y   dy  
e.g. (i)  y     order 2 and degree 4
dx 2   dx  

dy 1
y 
(ii) dx dy order 1 and degree 2
dx

d3 y 2
(iii) e dx3  x d y2  y  0 order 3 and degree not defined
dx

1  dy  dy
(iv) sin    xy   sin(x  y) order 1 and degree 1
 dx  dx

 dy  dy
(v) ln   = ax + by  = eax + by order 1 and degree 1
 dx  dx
Formation of differential equation
If an equation in independent and dependent variables having some arbitrary
constant is given , then a differential equation is obtained as follows :
Step 1 : Differentiate the given equation say f(x,y,c1) = 0 w.r.t. the independent
variable (say x) as many times as the number of arbitrary constants in it.
Step 2 : Eliminate the arbitrary constants.
The eliminant is the required differential equation .
Note: A differential equation represents a family of curves all satisfying some common
properties. This can be considered as the geometrical interpretation of the differential
equation.

Page # 2
DIFFERENTIAL EQUATION

Ex.1 Find diffrential equation of all concentric circle at the origin


Sol. Equation of such circles is x2 + y2 = r2
differentiating wrt x we get
 xdx + ydy = 0
(order is 1 and degree also 1)
Ex.2 Find diffrential equation of all circles touching the x axis at the origin and centre
on y-axis.
Sol. Equation of such circles is
x2 + (y – a)2 = a2
x2 + y2 – 2ay = 0 ....(1)

dy dy dy dy
differentiating , 2x + 2y = 2a or x+y =a
dx dx dx dx
substituting the value of ‘a’ in (1)

dy
(x2 – y2) = 2xy (order is one again and degree 1)
dx
Ex.3 Form the diffrential equation of the family of parabolas with focus at the origin and
axis of symmetry along the x-axis.
Sol. Equation of the parabola is
(x2 + y2 ) = (x + 2A)2
y2 = 4A ( A + x)
dy dy
2y  4A y  2A
dx dx
2 2
 dy 
2 dy  dy  dy
Hence y   y   2y .x or y 2  y 2    2xy (order 1 and degree 2)
 dx  dx  dx  dx
Ex.4 Form the diffrential equation of family of lines situated at a constant distance p
from the origin.

Sol. Method I : y = mx + p 1  m2 (Tangents to the circle x2 + y2 = p2 )

dy
where m =
dx
Note that order is 1 and degree is 2.
Method II : Consider the line x cos  + y sin  = p where p is a constant and 
varies. Diffrentiate w.r.t. x i.e. cos  + y ' sin  = 0  y ' = – cot  . Put in the given
line.
General and Particular Solutions
The solution of a differential equation which contains a number of independent
arbitrary constants equal to the order of the differential equation is called the
General solution . A solution obtainable from the general solution by giving particular
values to the constants is called a Particular solution.
Note that the general solution of a differential equation of nth order contains ‘n’ &
only ‘n’ independent arbitrary constants. The arbitrary constants in the solution of
a differential equation are said to be independent, when it is impossible to deduce
from the solution an equivalent relation containing fewer arbitrary constants. Thus
the two arbitrary constants A, B in the equation y = A ex + B are not independent

Page # 3
DIFFERENTIAL EQUATION

since the equation can be written as y = A eB. ex = Cex. Similarly the solution y = A
sin x + B cos (x + C) appears to contain three arbitrary constants, but they are really
equivalent to two only . Similarly y = (C1 + C2)cos ( x + C3) – C 4 e x  C5 has only 3
independent arbitrary constants hence will be of order 3.

Types of first order and first degree differential equations


1. Variable-Seperable form of Differential Equation : If the differential equation
can be expressed as ; f(x) dx + g(y) dy = 0 then this is said to be variable -
separable type .

A general solution of this is given by  f(x) dx +  g(y) dy = c ;

where c is the arbitrary constant.


dy
Ex.5 Solve ln  ax  by
dx

dy dy ax  by  by
Sol. ln  ax  by  e = eax . eby or e dy   eax dx
dx dx
Ex.6 Find the nature of the curve not passing through the origin for which the length of
the normal (tangent) at the point P is equal the radius vector of the point P.
1
Sol. Equation of normal : Y – y =  (X –x)
m
ON2 = GN2

x2 + y2 = m2 y2 + y2

x
 m=+ y

with +ve, xdx – ydy = 0


x2 – y2 = C (Rectangular Hyperbola)
with –ve x2 + y2 = C (circle)
|||ly we can do for tangent where Y– y = m (X – x)
[Ans: straight line or rectangular hyperbola]

2. Differential equation Reducible to Variable - Seperable Form :

dy
d x = f (ax + by + c) , b  0. (If b = 0 this is directly variable separable)
To solve this , substitute t = ax + by + c . Then the equation reduces to
separable type in the variable t and x which can be solved.

dy
Ex.7 Solve (x – y)2  1.
dx

dz dy dy dz
Sol. Put z = x –y  1  1
dx dx dx dx

Page # 4
DIFFERENTIAL EQUATION
 dz  2 dz
Now z2 1   1 z  z2 1
 dx  dx

z2
 dx = dz , which is in the form of variable separable
z2 1

1 z 1
Now integrating, we get x = z + ln c
2 z 1

1 x  y 1
 Solution is x =(x – y) + 2 ln x  y  1  1

DRILL I
1. Find the order and degree of the differential equation

3 5/3
5 4
 d2 y   dy  d 2 y   dy  
(a)  2   3   2y  x sin x (b)  1    
 dx   dx  dx 2   dx  

 (y ')2 (y ')3  d2 y  dy 
(c) y  ln 1  y '   ......... (d)  x ln  
 2! 3!  dx 2  dx 

d 2 y  dy   d2 y  d2 y  dy 
(e)     ln  2 (f)  cos  
dx 2  dx   dx  dx 2
 dx 

dy dy
2. 1  x 2  y 2  x 2 y 2  xy 0 3. xy 2  1  x2  y2  x 2y2
dx dx

1  dy  dy dy
4. (i) sin  xy (ii)  cos(x  y) (iii)  (4x  y  1)2
 dx  dx dx

dy
5. (x  y)2  a2 6. cos (x + y) dy = dx.
dx

dy
7.  cos(x  y)  sin(x  y) 8. (x2 + 4y2 + 4xy) dy = (2x + 4y + 1) dx.
dx

 x  y  a  dy x  y  a dy
9.    10.  x tan(y  x)  1 .
 x  y  b  dx x  y  b dx

dy xy
11.  (x  y  1) 
dx log(x  y)
12. Solve the differential equation by variable separable method:
(ey + 1) cos x dx + ey sin x dy = 0.
Solve the following differential equations by reducing them to variable separable
form:

 x  y  1  dy x  y  1
13.   
 x  y  2  dx x  y  2

Page # 5
DIFFERENTIAL EQUATION
dy
14. (3x  4y  5)2  a2
dx

dy x  y  1
15. 
dx x  y  1
16. Find the differential equation of all circles touching the
(i) x-axis at the origin (ii) y-axis at the origin
17. Obtain the differential equation of all circle of radius r.

18. Find the differential equation of all the circles in the first quadrant which touch
the coordinate axes.
3. Homogeneous Differential Equation
dy f (x , y)
A differential equation of the form d x =  (x , y) , where f (x , y) & f (x , y) are

homogeneous functions of x & y , and of the same degree , is called homogeneous.


dy x
This equation may also be reduced to the form d x = g   & is solved by putting y =
y
vx so that the dependent variable y is changed to another variable v , where v is
some unknown function, the differential equation is transformed to an equation
with variables separable.

dy y (x  y)
Consider d x + = 0.
x2
Important Notes :
(a) The function f (x , y) is said to be a homogeneous function of degree n if
for any real number t (  0) , we have f (tx , ty) = tn f(x , y).
e.g. f(x , y) = ax2/3 + hx1/3 . y1/3 + by2/3 is a homogeneous function of degree 2/3.

dy
(b) A differential equation of the form d x = f(x, y) is homogeneous if f (x , y) is a

homogeneous function of degree zero i.e. f (tx , ty) = t f (x , y) = f(x , y). The
y x
function f does not depend on x & y separately but only on their ratio x or y .

dy x 2  y 2
Ex.8 Solve  .
dx 2xy

dy x 2  y 2
Sol.  (homogeneous ) . Put y = vx
dx 2xy

x2  v2x2 dv
 vx
2x.vx dx

dx 2v
  dv , Integrate
x 1  v2
C + lnx = - ln(1 –v2)
 ln kx + ln(1 –v2) =0  kx(1- v2) = 1  k(x2 – y2) = x .

Page # 6
DIFFERENTIAL EQUATION

 dy  y
Ex.9 Solve the differential equation  x  y  tan1 = x given y x 1  0
 dx  x

 dy y  y  dy y  y
Sol. x   .tan1  x or    tan1 = 1
 dx x  x  dx x  x

dy dv  dv 
put y = vx  v  x or  v  x  v  tan1 v  1
dx dx  dx 

dv dx
x tan1 v  1 or  tan
1
v dv  
dx x

v
v tan–1v – 1 v 2
dv = lnx + C

1
v. tan–1v – ln(1 + v2) = lnx + C
2
where x = 1 ; y =0  v = 0  C = 0

y y y2 y2
tan1  ln 1  2  ln x = ln x 1  2 = ln x2  y2
x x x x
y y
tan1
 x2  y 2  e x x

Sometimes a Homogeneous Diffrential equation is given in the form of a quadratic


dy
equation in ( i.e. degree 2, consider the example)
dx

Ex.10 Find the curve such that the angle, formed with the x-axis by the tangent to the
curve at any of its points, is twice the angle formed by the polar radius of the point
of tangency with the x-axis. Interpret the curve.
dy y
Sol. Given tan2  = where tan  =
dx x

2tan  dy
2

1  tan  dx

y
2
dy x  2xy  1  v2 1  v2  2v 2 
or = y 2 2
x y 2 
2
dv   2
dv 
dx 1 2  v(1  v ) v(1  v ) 
x
[Ans : x2 + y2 – 2cy = 0]
Note : That the curve is a circle with centre on y-axis and touching the x-axis at the
origin.
Ex.11 Find the curve such. that the ratio of the subnormal at any point to the sum of its
abscissa and ordinates is equal to the ratio of the ordinate of this point to its
abscissa. If the curve passes through (1, 0) find all possible equations in the form
y = f (x).

Page # 7
DIFFERENTIAL EQUATION

1
Sol. Normal : Y – y =  ( X – x)
m

|my| y dy xy
  
xy x dx x
with +ve sign solution is y = x ln | cx |
wit h –ve sign solu t ion is x 2 + 2xy = c
Ex.12 Find the curve for which the sum of the normal and subnormal is proportional to
the abscissa, the proportionality constant being k.
Sol. Refer to the figure of

m2 y 2  y 2  (my)  kx

|y| 1  m2  kx – |my|

squaring y2 (1+m2) = k2x2 + m2 y2 – 2kx my


 2kx |my| = k2 x2 – y2

dy k 2x2  y 2

dx 2kxy

4. Differential Equation Reducible to Homogeneus


dy a1x  b1 y  c1 a1 a2
If d x = a x  b y  c ; where a1b2 - a2b1  0 and a 2  b1  0 , i.e. b  b
2 2 2 1 2

then the substitution x = u + h, y = v + k transform this equation to a


homogeneous type in the new variables u and v where h and k are arbitrary
constants to be chosen so as to make the given equation homogeneous

dy 2x  y  1
Ex.13 = 6x  5 y  4
dx
Sol. x = u + h ; y = v + k ; dx = du ; dy = dv

dy dv

dx du

dv 2(u  h)  (v  k)  1 2u  v
  
du 6(u  h)  5(v  k)  4 6u  5v
where 2h – k + 1 = 0 and 6h – 5k + 4 = 0
dv dz
put v = u z  u z
du du

v
2
dz 2z 2
u +z= =
du 6  5z v
65
u
u dz 2z
  z
du 6  5z

Page # 8
DIFFERENTIAL EQUATION

DRILL - II
Solve the following differential equation
1. (3xy + y2) dx + (x2 + xy) dy = 0. 2. (x3 – 3xy2) dx = (y3 – 3x2y) dy.

 y  y   y  y 
3. y  x cos    y sin    dx  x  y sin    x cos    dy  0 .
 x  x   x  x 

y y
4. y cos   (x dy  y dx)  x sin   (x dy  y dx)  0 .
x x

x/y x/y  x dy x 2  y 2  2x  2y  2
5. 1  e  dx  e 1   dy  0
y 6.
dx

xy  x  y  1
.

 y  2
(x  1)2  (y  2)2 tan1  
dy  x 1  x   x  
 xy log   dx   y 2  x 2 log    dy  0
7. dx  y  2 . 8.
(xy  2x  y  2)tan1   y   y  
 x 1 
dy 2x  y  3 dy x  y  1
9.  10. dx  x  y  1
dx x  2y  4 .

dy 2y  x  4 dy cos x(3cos y  7sin x  3)


11. 
dx y  3x  3 12. dx  sin y(3sin x  7cos y  7)  0 .

2
dy  x  2y  3 
13. Find the general solution of the differential equation:  
dx  2x  y  3 

dy
14. (2x  4y  3)  2y  x  1
dx
15. Show that the differential equation that represents all parabolas having their axis
of symmetry coincident with the axis of x is yy 2  y12  0
16. Find the differential equation of all non-horizontal lines in a plane.

5. Linear Differential Equation


A differential equation is said to be linear if the dependent variable & all its
differential coefficients occur in degree one only and are never multiplied together.
The nth order linear differential equation is of the form ;

dn y d n1 y
a0 (x) + a1(x) + ...... + an (x) . y = f (x) . Where a0(x) , a1(x) ..... an(x) are
d xn d x n1
called the coefficients of the differential equation .
Note that a linear differential equation is always of the first degree but
every differential equation of the first degree need not be linear.
3
d2 y  d y 
e.g. the differential equation   + y2 = 0 is not linear , though its
d x2  d x 
degree is 1.

Page # 9
DIFFERENTIAL EQUATION
Linear Differential Equation of first order :
The most general form of a linear differential equations of first order is

dy
d x + Py = Q, where P & Q are functions of x . ( Independent variable)

To solve such an equation multiply both sides by e  P d x (Integrating factor)


Note :

(1) The factor e


Pdx on multiplying by which the left hand side of the
differential equation becomes the differential coefficient of some function
of x & y, is called integrating factor of the differential equation popularly
abbreviated as I. F.
(2) It is very important to remember that on multiplying by the integrating factor,
the left hand side becomes the derivative of the product of y and the I.F.
(3) Some times a given differential equation becomes linear if we take y as
the independent variable and x as the dependent variable. e.g. the equation;
dy dy x  2y 3 1
(x + 2y3)
= y can be written as = = x + 2y2, which is a
dx dx y y
linear differential equation.
1
dy y etan x
Ex.14 Solve the differential equation   .
dx (1  x 2 ) 1  x 2
1
dy y etan x
Sol.  
dx (1  x 2 ) 1  x 2
Multiplying both sides by I.F. and integrating
1
tan1 x etan x tan1 x
ye   e dx  c
(1  x 2 )
1

tan 1
x etan x
Put e t dx  dt
1  x2

1 t2
 ye tan x
 tdt  c 
 c
2
1 1
 2ye tan x
 e2 tan x
 2c

Ex.20 Solve the differential equation t(1+ t2) dx = (x + xt2 – t2 ) dt; x t 1  
4
dx x(1  t2 )  t2 x t
Sol.  2
 
dt t(1  t ) t 1  t2

1
dx 1 t  dt 1
 x ; I.F. e t  e ln t 
dt t 1  t2 t

1 dx 1 1 d 1  1
 2x   .x   
t dt t 1  t2 dt  t  1  t2

Page # 10
DIFFERENTIAL EQUATION

x dt
  = – tan–1t + c  x = t (c – tan–1 t)
t 1  t2


t=f ;x=–  c=0 [Hence x = – t tan–1t]
4

dy
Ex.16 Solve the differential equation x (x2+ 1) = y(1– x2 ) + x2 lnx
dx

dy y(1  x 2 ) x ln x dy (1  x 2 )y x ln x
Sol.  2
 2 
 
dx x(1  x ) 1  x dx x(1  x ) 1  x 2
2

1
1
x2
x 2 1  1  1
dx
 x(x 2 1) x ln  x   1
I.F. e
x
= e x
= e 
= x
x

 x2  1  x ln x x 2  1  x2  1 
 y .   . dx    y  ln x dx + c 
 x  x2  1 x  x 

dy
Ex.17 Solve the differential equation x lnx + y = 2 lnx
dx

dy
Sol. x ln x  y  2ln x
dx

dy 1 2 1
 y I.F.  x ln x dx
dx x ln x x e  eln(ln x )  ln x

dy 1 2
 ln x  y  .ln x
dx x x

d 2
  y.ln x   ln x
dx x

ln x
y lnx = 2  dx  c [Ans. y ln x = ln2 x + c ]
x
Ex.18 A tank initially contains 100 litres of brine in which 50 gms of salt dissolved. A brine
containing 2 gm/litre of salt runs into the tank at the rate of 5 litre/min. The
mixture is kept stirring and flows out of the tank at the rate of 4 litres/min then
(a) At what rate (gms/min) does salt enter the tank at time t.
(b) What is the volume of the brine in the tank at time t.
(c) At what rate (gms/min) does salt leave the tank at time t.
(d) form the DE of the process and solve it to find an expression for the amount of
salt present at time t.
2gm 5 litres
Sol. (a) Rate at which salt is added = · = 10 gm/min.
litre min
(b) Volume V (t) of brine at time t = 100 + (5 – 4) t = (100 + t) litres

y(t) y
(c) Rate at which salt leaves = (out flow rate) = ·4
V(t) 100  t

Page # 11
DIFFERENTIAL EQUATION

dy 4y dy 4y
(d) Hence DE is = 10 – which is linear + = 10
dt 100  t dt 100  t
4

 dt
I.F. = e 100  t = e+4(ln (100 + t)
4
 y (100 + t)4 = 10 (100  t) dt = (100 + t)4 + c

y (100 + t)4 = 2 (100 + t)5 + c
if t = 0 ; y = 50
50 · 1004 = 200 · 1004 + c
 c = – 150 · 1004
 y (100 + t)4 = 2 (100 + t)5 – 150 · 1004

150·1004
y = 2 (100 + t) – ;
(100  t)4

150
y = 2 (100 + t) – 4
 t 
1  
 100 
Ex.19 A storage tank contains 2000 litres of gasoline which initially has 100 gms of an
additive dissolved in it. Gasoline containing 2 gms of additive per litre is pumped
into the tank at a rate of 40 litre/min. The well mixed solution is pumped out at a
rate of 45 litre/min. Form the DE and express the amount of additive in gasoline as
a function of t.

dy y
Sol.  80  · 45
dt (2000  5t)
t = 0 ; y = 100
9
 t 
y = 10(400 – t) – 3900 1  
 400 

6. Differential Equation Reducible to Linear Form : (Bernoulli’s Equation)


dy
The equation d x + py = Q . yn where P & Q functions of x , is reducible to the

linear form by dividing it by yn & then substituting y-n+1 = Z. Its solution can be
obtained as in the normal case.

dy
Ex.20 Solve : d x = xy + x3y2

1 dy x 1 dy x
Sol. 2 =  x 3 or 2 – = x3
y dx y y dx y

1 1 dy dz
put – =z  2 =
y y dx dx

dz
 + z·x = x3 which is linear.
dx

Page # 12
DIFFERENTIAL EQUATION

DRILL - III
dy dy y sin x
1.  y  cos x  sin x 2.   cos x 
dx dx x x
3. Solve y dx – (x + 2y2) dy = 0. 4. Solve y dx + (x – y3) dy = 0.

5. Solve (x + 2y3) dy = y dx.


dy
6. If y1 and y2 are the solutions of the differential equation  Py  Q , where P and Q
dx
Q
 y1 dx
are functions of x alone and y2= y1z, then prove that z  1  Ce , where C is an
arbitrary constant.

dy
7. If y1, y2 are two solutions of the differential equation  P(x).y  Q(x) , then prove
dx
that y=y1 + C (y1 – y2) is the general solution of the equation where C is an arbitrary
constant. For what relation between the constants a, b will the linear combination
ay1 + by2 also be a solution ?
dy dy
8. x  y  y 2 log x 9.  x 3 y 3  xy
dx dx

dy dy
10. tan y  tan x  cos y.cos3 x 11.  x sin2y  x 3 cos2 y
dx dx

dy dy y y
12. 3x(1  x 2 )y 2  (2x 2  1)y 3  ax 3 13.  log y  2 (log y)2
dx dx x x

dy 1
14. 
dx xy[x sin y 2  1]
2

dy
15.  y ln2  2sin x (cos x  1)ln 2 , y being bounded when x  + 
dx

dy
15.  y ln2  2sin x (cos x  1)ln 2 , y being bounded when x  + 
dx

 sin4 
1  f() 
 3
16.   d where
2  4 (f(cos 2  ))  f()   = x + y and <x+y<
 cos   2 4

Clairaut’s Equation
The differential equation

dy
y = mx + f(m), ..............(1), where m =
dx
is known as Clairaut’s Equation.
To solve (1), differentiate it w.r.t. x, which gives

Page # 13
DIFFERENTIAL EQUATION
dy dm d f(m) dm
=m+x +
dx dx dm dx

dm d f(m) dm
x + =0
dx dm dx

dm
either = 0  m = c ...........(2)
dx
or x + f ' (m) = 0 ............(3)
Note :
(i) If m is eliminated between (1) and (2), the solution obtained is a general
solution of (1)
(ii) If m is eliminated between (1) and (3), then solution obtained does not
contain any arbitrary constant and is not particular solution of (1). This
solution is called singular solution of (1).

Orthogonal Trajectory :
An orthogonal trajectory of a given system of curves is defined to be a curve which
cuts every member of a given family of curve at right angle.

Steps to find orthogonal trajectory :


(i) Let f (x, y, c) = 0 be the equation of the given family of curves, where 'c' is an
arbitrary constant.
(ii) Differentiate the given equation w.r.t. x and then eliminate c.

dy dx
(iii) Replace by – dy in the equation obtained in (ii).
dx
(iv) Solve the differential equation obtained in (iii).
Hence solution obtained in (iv) is the required orthogonal trajectory.
Ex.21 Solve y2 = 4a (x + a)
Sol. y2 = 4a (x + a)
2 y y1 = 4a ....(1)

 yy1 
y2 = 2 y y 1  x 
 2 
y2 = 2x y y1 + y2 · y12
2 2
 dy   dy   dx   dx 
y2   + 2xy  dx  – y2 = 0  y  dy  – 2x  dy  – y = 0
 dx       

dx 2x  4x 2  4y 2 x  x2  y2
dy = 2y
=
y

dx dv
Let x = v y  dy = v + y
dx

dv
v+y =v± 1  v2  y = 0
dy

Page # 14
DIFFERENTIAL EQUATION

ln (v + 1  v2 ) = ± ln(cy)

x  x2  y2
= cy  x + x 2  y 2 = cy2
y

Exact Differential Equation


dy
The differential equation M + N = 0 ...........(1)
dx
Where M and N are functions of x and y is said to be exact if it can be derived by
direct differentiation (without any subsequent multiplication, elimination etc.)
of an equation of the form f(x, y) = c

dx
e.g. y2 dy + x dx + = 0 is an exact differential equation.
x
Following Exact Differentials Must Be Remembered :

xdy  ydx y


(i) xdy + y dx = d(xy) (ii) 2
 d 
x x

ydx  xdy x xd y  ydx


(iii)  d  (iv)  d(ln xy)
y 2
y xy

dx  dy xdy  ydx  y
(v)  d  ln 
x  y = d (ln (x + y)) (vi)
xy  x

ydx  xdy  x xdy  ydx  y


(vii)  d  ln  (viii) 2 2
 d  tan1 
xy  y x  y  x

ydx  xdy  x xdx  ydy


(ix) 2 2
 d  tan1  (x) 2 2
 d ln x 2  y 2 
x  y  y x y  

 1  x dy  y dx  e x  y ex dx  ex dy
(xi) d   d
(xii)  y  
 xy x2 y2   y2

 e y  x e y dy  e y dx
(xiii) d  x  
  x2

DRILL - IV
Solve the following differential equations
1. x dy + y dx + 2x3 dx = 0. 2. x dx + y dy = a(x2 +y2) dy

3. Solve: (1 + xy) x dy + (1 – xy) y dx = 0.


4. Solve: y dx – x dy + (1 + x2) dx + x2 sin y dy= 0.

5. (1  x x 2  y 2 ) dx  (1  x 2  y 2 )y dy  0 .

y  y 
6. ye x/y dx  (xe x/y  y 2 sin y) dy . 7. x sin   dy   y sin    x  dx .
x  x 

Page # 15
DIFFERENTIAL EQUATION

x dx  y dy 1  x2  y 2  x
8.  . 9. (1  ex/y ) dx  e x/y 1   dy  0 .
x dy  y dx x2  y2  y

dy
xy 2 2 2
dx  x sin (x  y ) x dy  y dx
10. dy y3 11. x dx  y dy  0.
yx x2  y 2
dx

dy 3x 2 y 4  2xy
12.  2 .
dx x  2x 3 y 3

13. (3 tan x  4 cot y  7)sin2 y dx  (4tan x  7cot y  5)cos 2 x dy  0

x
14. cos   (y dx  x dy)  xy 3 (x dy  y dx) . 15. x dy – {y + xy3(1 + log x)} dx = 0.
y

16. (x2 – 4xy – 2y2) dx + (y2 – 4xy – 2x2) dy = 0.


17. 1  xy  ydx  1  xy  xdy  0 18. xdx  ydy  m  xdy  ydx 

dy
xy 2 2 2
 x2  dx  x sin (x  y )
19. (2x ln y)dx    3y 2  dy  0 20. dy y3
 y  yx
dx

xdx  ydy  a 2  x2  y2 
21. xdy  ydx
  2 2 
 x y 
22. Find the orthogonal trajectories of the familiy of parabolas y2 = 4ax, where ‘a’ is the
variable parameter
23. Find the orthogonal trajectories of the family of rectangular hyperbola xy = c2

DRILL - V
(Word Problems)
1. Show that the curve for which the normal at every point passes through a fixed
point is a circle.

y y y
2. The slope of the tangent to a curve at any point (x, y) on it is given by  cot cos
x x x
(x > 0, y > 0) and the curve passes through the point (1,  /4). Find the equation of
the curve.
3. If the tangent at any point P of a curve meets the axis of X in T. Find the curve for
which OP = PT, O being the origin.
4. Determine the equation of the curve passing through the origin in the form y = f(x),
dy
which satisfies the differential equation  sin(10x  6y) .
dx

Page # 16
DIFFERENTIAL EQUATION

5. Find the curve for which portion of the x – axis cut off between the origin and the
tangent at a. point is twice the absicissa and which passes through the point (1, 2).
6. Find the curves for which the portion of the y – axis cut off between the origin and
the tangent varies as the cube of the abscissae of the point of contact.
7. Find the curve for which the length of the perpendicular from the foot of the ordinate
of any point on the curve on the tangent is constant, equal to C and also given that
the curve cuts the axis of y at right angles.
8. Show that the curve such that the ratio of the distance between the normal at any
of its points and the origin to the distance between the same normal and the point
(a, b) is equal to the constant k (k > 0, k  1) is a circle.
9. A normal is drawn at a point P(x, y) of a curve. It meets the x – axis at Q. If PQ is of
constant length k, then show that the differential equation describing such curves
dy
is y   k 2  y 2 and find the equation of such a curve passing through (0, k).
dx
10. A curve passing through the point (1, 1) has the property that the perpendicular
distance of the normal at any point P on the curve from the origin is equal to the
distance of P from x- axis. Determine the equation of the curve.
11. Let y = f(x) be a curve passing through (1, 1) such that the triangle formed by the
coordinate axes and the tangent at any point of the curve lies in the first quadrant
and has area 2. Form the differential equation and determine all such possible
curve.
12. A curve C has the property that the area of the triangle formed by the x – axis,
tangent to the curve and radius vector of the point of tangency is constant k2. Find
the equation of the curve C if the point (0, 1) lies on the curve.
13. Find the equation of the curve which is such that the area of the rectangle
constructed on the abscissa of any point and the initial ordinate of the tangent at
this point is a constant equal to a2.
14. Find a pair of curves such that,
(i) The tangents at points with equal abscissae intersection on y – axis.
(ii) The normals drawn at points with equal abscissae intersect on x – axis.
(iii) One curve passes through (1, 1) and the other passes through (2, 3).
15. A and B are two separate reservoirs of water. Capacity of reservoir A is double the
capacity of reservoir B. Both the reservoirs are filled completely with water, their
inlets are closed and then the water is released simultaneously from both the
reservoirs. The rate of flow of water out of each reservoir at any instant of time is
proportional to the quantity of water in the reservoir at that time one hour after the
1
water is released, the quantity of water in reservoir A is 1
times the quantity of
2
water in reservoir B. After how many hours do both the reservoirs have the same
quantity of water ?
16. the rate at which a substance cools in moving air is proportional to the difference
between the temperatures of the substance and that of the air. If the temperature
of the air is 290°K and the substance cools from 370°K to 330°K in 10 minutes,
when will the temperature be 295°K

Page # 17
DIFFERENTIAL EQUATION

17. The population of a country increases ata rate proportional to the number of
inhabitants. If the population doubles in 30 years, find after how many years the
population will triple ?
18. A right circular cone with radius R & height H contains a liquid which evaporates
at a rate proportional to its surface area in contact with air (proportionality constant
K > 0). Find time after which the cone is empty.

SOLVED EXAMPLES
Ex.1 Find the differential equation of the family of curves y = Ae x + Be3x for different
values of A and B.
Sol. y = A ex + Be3x . . . . (1)

 dy 
y1 = Aex + 3Be3x . . . (2)  y1  
 dx 

 d2 y 
y2 = Ae + 9B x 3x
. . . (3)  y 2  
 dx 2 

Eliminating A and B from the above three, we get

ex e3x y 1 1 y
x 3x
e 3e  y1 1 3  y1
= 0  ex e3x =0
x 3x
e 9e y2 1 9 y2

Ex.2 Find order of the differential equation, whose general solution is

y = C1 ex + C2 e2x + C3 e3x + C4 e x c5 , where C1, C2, C3, C4, C5 are arbitrary constants.

Sol. y = (c1 + c4) ex + c2 e2x + c3 e3x + c4 e c5


c5
y = c1 ex + c2 e2x + c3 e3x + c4 e
y = k1 ex + k2 e2x + k3 e3x + k­4
Therefore 4 obitrary constants
Hence order is 4.
Ex.3. Find solution of the differential equation (1 – xy – x5y5) dx – x2(x4y4 + 1)dy = 0
Sol. The given equation is dx – x (ydx+ xdy )= x5y4(ydx + xdy)
dx 1
  (1  x 4 y 4 )d(xy)  ln x  xy  x 5 y 5  lnc
x 5
1
xy  x5 y 5
 x = ce. 5

Ex.4 A tank contains 200 litres of brine in which 20 gms of salt dissolved. Brine containing
1
gm of salt/lit. runs into the tank at the rate of 2 litre/min. The mixture is kept
4
stirring runs out at the same rate. Express the concentration of solution in tank in
grams as a function of time t. What is the limiting value approached by the amount
of salt as t   . Whenever was the amount of salt in solution be 20 gm.

Page # 18
DIFFERENTIAL EQUATION

Sol. t = 0; y = 20

dy 1 y dy 1 y 50  y
  ·2    
dx 2 200 dx 2 100 100

dy dt
Hence 50  y  100

t
– ln (50 – y) = +c
100

t
or ln (50 – y) = – +c
100
t = 0 ; y = 20 ; c = ln 30

50  y t
Hence ln =
30 100
t t
50  y  
= e 100
 y = 50 – 30· e 100
30
t

[Ans. (i) y = 50 – 30 · e 100
, (ii) y when t   = 50 gms, (iii) y > 20 for all t > 0 ]

Ex.5 Given a function ‘g’ which has a derivative g ' (x) for every real ‘x’ and which satisfy
g ' (0) = 2 and g(x + y) = ey . g(x) + ex . g(y) for all x & y.
Find g(x) and determine the range of the function. Also compute the area bounded
by f (x) and the x axis.
Sol. Put x = 0 ; y = 0  g (0) = 0

g (h)
and g ' (0) = Limit = 2
h0
h

g (x  h)  g (x)
Now g ' (x) = Limit
h0
h

e h . g (x)  e x . g (h)  g (x)


= Limit
h0
h

 eh  1  g (h)
= g (x) Limit   + ex Limit or g ' (x) = g (x) + 2 ex
 h  h
h0 h  0

dy dy
Let g (x) = y then d x = y + 2 ex  e -x d x - y e -x = 2

d x  0
-x -x
 d x (y e ) = 2 y e = 2 x + c ; If y  0   c = 0

Hence y = 2 x ex = g (x)

dy
Now x x
dx = 2 [ e + x e ] = 0  x = -1

 minima at x = - 1

Page # 19
DIFFERENTIAL EQUATION

 2 
 range is   ,  
 e 

 2 
[Ans: g(x) = 2x ex,   ,   ; 2 sq. units ]
 e 
x
y(u)
Ex.6 The function y (x) satisfies the equation y (x) + 2x 1 u 2
du = 3x2 + 2x + 1. Show that
0

x
y(u)
the substitution z (x) = 1 u 2
du converts the equation into a first order linear
0

differential equation for z(x) solve for z (x) . Hence solve the original equation for y (x).
x
y(u)
Sol. y (x) + 2x 1 u 2
du = 3x2 + 2x + 1 ....(1)
0

x
y(u)
Let z (x) = 1 u 2
du
0

y(x)
 z ' (x) =  y (x) = (1 + x2) z ' (x) ....(2)
1  x2
substituting y (x) in equation (1)
(1 + x2) z ' (x) + 2x · z(x) = 3x2 + 2x + 1
dz
Let z ' (x) =
dx

dz 2x 3x 2  2x  1
+ z (x) = which is linear DE
dx 1  x2 1  x2
2x
I.F. = e  1 x 2 dx = eln(1 x 2 ) = 1 + x 2

2
Hence z (1 + x 2) =  (3x  2x  1)dx

z (1 + x 2) = x3 + x2 + x + C
but z = 0 when x = 0  C = 0

x3  x 2  x
Hence z (x) = .....(3)
1  x2
now y (x) = (1 + x 2)·z ' (x) .....(4) (from 2 )

x(1  x 2 )  x 2 x2  1  1 1
also z (x) = = x+ = x +1–
1  x2 1 x 2
1  x2

2x (1  x 2 )  2x
z ' (x) = 1 + =
 (1  x 2 )2 (1  x 2 )2

(1  x 2 )2  2x
 y (x) = [ from eq n (4) ]
(1  x 2 )

Page # 20
DIFFERENTIAL EQUATION

Ex.7 In an equation of the form : y f (xy) dx + xg (xy) dy = 0 the variables can be separated
by the substitution xy = v. Consider the example
(x3 y3 + x2y2 + xy + 1 ) y dx + (x3y3 – x2y2 – xy + 1) x dy = 0
v
Sol. xy = v  y=
x
dy dv
x y = ....(1)
dx dx

dy
now equation become y.(v3 + v2 + v + 1) + (v3 – v2 – v + 1) x. =0
dx

 dv 
y.(v3 + v2 + v + 1) + (v3 – v2 – v + 1)   y = 0
 dx 

dv 3
y. [ (v3 + v2 + v + 1) – (v3 – v2 – v + 1) ] + (v – v2 – v + 1) = 0
dx

v dv 3
(2v2 + 2v) + (v – v2 – v + 1) = 0
x dx

2v 2 (v  1) dv 3 dx v3  v 2  v  1
 2
(v  v  v  1) = 0  2    dv
x dx x v2 (v  1)

dx (v 2  1)(v  1) dx v2  2v  1
2    dv or 2 dv
x v2 (v  1)  x
  v2
Ex.8 Solve (x cos y – y sin y) dy + (x sin y + y cos y) dx = 0
Sol. Let x siny + y cosy = t
diff. w.r.t. x
dy dy dy dt
x cosy + siny + cosy – y siny =
dx dx dx dx
dy dy dt
(x cosy – y siny) + siny cosy =
dx dx dx
dt dy
– (siny + cosy )+t=0
dx dx
dy dz
siny = z ; cosy =
dx dx
dt  dz  d
– z   + t = 0 or (t  z) + (t – z) = 0
dx  dx  dx
Let t–z=u

du
+ u = 0  lnu + x = C  ln (t – z) + x = C
dx
ln (x siny + y cosy – siny) = C – x  x siny + y cosy – siny = C e–x

y x
Ex.9 Misleading appearance of a D. E. containing terms like & y , however may not be
x
homogeneous and this can not be solved by substituting y = v x.

Page # 21
DIFFERENTIAL EQUATION

1 x y y  1 y x x 1 
e.g.  sin  2 cos  1 dx +  x cos x  2 sin y  2  dy = 0
y y x x   y y 

x y
Sol. Isolating sin & cos
y x

 dx x dy  x  dy y dx  y dy
  2  sin +   2  cos  dx  2 = 0
 y y  y  x x  x y

ydx  xdy x
2
sin + xdy 2ydx .cos y + dx + dy = 0
y y x x y2

x x y y
sin .d   + cos .d   + dx + dy = 0
y y x x y2

x y 1 1 x y
 cos  sin + x – = C or y  cos y  x  sin x
y x y
However note that the D.E.

 y y  y y
 x cos  y sin  ydx +  x cos  y sin  xdy = 0 is Homogeneous and can be
 x x  x x
solved conviniently with y = vx

2dx cos v  v sin v


simplifies to  =   dv
 x v cos v

y
 ln ( v cosv) + ln x2 = 0  xy cos =C
x

1 y2   x2 1
Ex.10 Solve : x
  2  dx +  2
  dy = 0
 (x  y)   (x  y) y 

dx dy x 2 dy  y 2 dx dx dy x 2 dy  y 2 dx
Sol.    0 or   2
0
x y (x  y)2 x y 2 21 1
x y   
y x

dy dx
 2
dx dy y2 x
or   2
0
x y 1 1
  
y x
y  sin x cos 2 (xy) x dy
Ex.11 Solve dx + + sin y dy = 0
2
cos (xy) cos2 xy

x dy  y dx
Sol. + sin x dx + sin y dy = 0
cos2 xy

d(xy)
 cos 2
 sin x dx  sin y dy  0
 
xy
tan (x y) = cos x + cos y + C

Page # 22
DIFFERENTIAL EQUATION
Ex.12 The force of resistance encountered by water on a motor boat of mass 'm' going in still
water with velocity 'v' is proportional to the velocity v. At t = 0 when its velocity is v0,
the engine is shut off. Find an expression for the position of motor boat at time t and
also the distance travelled by the boat before it comes to rest. Take the proportionately
constant as k>0.
dv
Sol. The resistance force oppositing the motion = m × acceleration = m .
dt

dv dv k
Hence DE is, m = – kv  =– dt
dt dt m
integrating
k
 ln v = – ·t + c
m
at t = 0, v = v0, Hence c = ln v0

v k
 ln v = – ·t
0 m
k
 ·t
v = v0 e m ....(1)
where v is the velocity at time t
k k
ds  ·t  ·t
now = v0 e m  ds = v0 e m dt
dt
kt
v0m 
Body's position at time t is, s (t) = – e m
+c
k

v0 m
if t = 0, s = 0  c =
k
kt
v 0 m 1  e  m 
 s (t) =   ....(2)
k  

mv0
To find how far the boat go, we have to find Lim s(t) =
t  k
k
v0 m   t
v m
Ans. S (t) = k  1  e m
 , distance moved = 0
  k
2
 dy  dy
Ex.13 Solve the differential equation y    2x  y  0; y x 0  5
 dx  dx

dy 2x  4x 2  4y 2 2
dy  x  x  y
2

Sol.  =  which is homogeneous


dx 2y dx y

dy dv
put y = vx  v  x
dx dx

dv x  x 1  v2 1  1  v2
vx = =
dx vx v

Page # 23
DIFFERENTIAL EQUATION

dv 1  1  v2 1  1  v2  v2
x = v =
dx v v
Taking +ve sign

v dv dx v dv dx
   or    x
2
v  1  (1  v ) 2 x 2
v 1  1  v 1
2

v
dv  dt
1  v2 1  t or
1  v2

dt dx
    ln tx = C
t x

 y2  
ln  1  2 1 x   C or ln  x 2  y 2  x   C
 x  
 
 

put x = 0 ; y = 5

ln  5  0  C  C = ln 5

 x2  y 2  5  x

x2 + y2 = 5 + x 2 + 2 5 x

 y2 = 5 + 2 5 x 
 y  5  2 5 x
|||ly with –ve sign, y2 = 5 – 2 5 x

Page # 24
DIFFERENTIAL EQUATION

EXERCISES

SINGLE CORRECT TYPE QUESTIONS


3/2
  dy 2 
1    
  dx  
1. The o r de r and de g r ee o f the d i f fe r e nti a l e q ua ti on r = are
d2 y
dx 2
respectively
(A) 2, 2 (B) 2, 3 (C) 2, 1 (D) none of these
2. The order of the differential equation whose general solution is given by
x  C5
y = (C1 + C2) sin (x + C3) – C4 e is
(A) 5 (B) 4 (C) 2 (D) 3
3. The order and degree of differential equation of all tangent lines to parabola
x2 = 4y is
(A) 1, 2 (B) 2, 2 (C) 3, 1 (D) 4, 1
4. If p and q are order and degree of differential equation
2 1/3
 d2 y   dy 
y 2
 2  + 3x   + x2y2 = sin x, then :
 dx   dx 

p 1
(A) p>q (B) q = (C) p = q (D) p < q
2
5. Family y = Ax + A3 of curve represented by the differential equation of degree
(A) three (B) two (C) one (D) none of these

dy
6. If = e–2y and y = 0 when x = 5, the value of x for y = 3 is
dx

e6  9
(A) e 5
(B) e + 1
6
(C) (D) loge 6
2
7. If f(x) = f ' (x) and f(1) = 2, then f(3) equals
(A) e2 (B) 2e 2 (C) 3e 2 (D) 2e 3

dy
8. If = 1 + x + y + xy and y (– 1) = 0, then function y is
dx
2
2 (1 x ) /2
(A) e(1 x) /2 (B) e 1 (C) loge (1 + x)–1 (D) 1 + x

x2  y2
9. Integral curve satisfying y ' = , y(1) = 2, has the slope at the point (1, 2) of
x2  y2
the curve, equal to

5 5
(A) – (B) –1 (C) 1 (D)
3 3

Page # 25
DIFFERENTIAL EQUATION
10. Solution of differential equation xdy – y dx = 0 represents :
(A) rectangular hyperbola
(B) straight line passing through origin
(C) parabola whose vertex is at origin
(D) circle whose centre is at origin
dy
11. The solution of the differential equation – ky = 0,y(0) = 1, approaches zero
dx
when x   , if
(A) k=0 (B) k > 0 (C) k < 0 (D) none of these

dv k
12. The solution of + v = – g is
dt m
k k
 t mg mg  m t
(A) v = ce m – (B) v = c – e
k k
k k
 t mg t mg
(C) v e m =c– (D) ve m =c–
k k
13. The differential equations of all conics whose centre lie at the origin is of order
(A) 2 (B) 3 (C) 4 (D) none of these
14. The differential equation for all the straight lines which are at a unit distance
from the origin is
2 2 2 2
 dy   dy   dy   dy 
(A) y x  = 1–   (B)  y  x  =1+  
 dx   dx   dx   dx 

2 2 2 2
 dy   dy   dy   dy 
(C) y x  =1+   (D)  y  x  = 1–  
 dx   dx   dx   dx 
15. The equation of the curve whose subnormal is constant a is
(A) y = ax + b (B) y2 = 2ax + b (C) ay2 – x2 = a (D) none of these

16. The differential equation of all conics whose axes coincide with the axes of
coordinates is of order
(A) 2 (B) 3 (C) 4 (D) 1

dy
17. If y1(x) is a solution of the differential equation + f(x) y = 0, then a solution of
dx

dy
differential equation + f(x) y = r (x) is
dx

1 r(x)
(A) y (x) dx (B) y1(x)  dx
y (x)  1 y1(x)

(C)  r(x)y (x) dx1 (D) none of these

Page # 26
DIFFERENTIAL EQUATION
dy
18. If y1(x) and y2(x) are two solutions of + f(x) y = r(x) then y 1(x) + y2(x) is solution
dx
of :

dy dy
(A) + f(x) y = 0 (B) + 2f(x) y = r(x)
dx dx

dy dy
(C) + f(x) y = 2 r(x) (D) + 2f (x) y = 2r(x)
dx dx
dy
19. The value of lim y(x) obtained from the differential equation = y – y2, where
x  dx
y(0) = 2 is
(A) zero (B) 1 (C)  (D) none of these
20. The slope of a curve at any point is the reciprocal of twice the ordinate at that
point and it passes through the point (4, 3). The equation of the curve is
(A) x2 = y + 5 (B) y2 = x – 5
(C) y2 = x + 5 (D) x2 = y + 5
21. The equation of the curve which is such that the portion of the axis of x cut off
between the origin and tangent at any point is proportional to the ordinate of
that point is
(A) x = y (b – a log y) (B) log x = by2 + a
(C) x2 = y (a – b log y) (D) none of these
(a is constant of proportionality)

22. The solution of y dx – x dy + 3x 2 y2 e x3 dx = 0 is

x x3 x x3
(A) + e =C (B) – e =0
y y

x x3
(C) – + e =C (D) none of these
y

dy 1  y2
23. The solution of + = 0 is
dx 1  x2
(A) sin–1 x, sin–1 y = C (B) sin–1 x = C sin–1 y
(C) sin–1 x – sin–1 y = C (D) sin–1x+sin–1 y = C
24. The solution of the differential equation y 1 y3 = 3y22 is
(A) x = A1 y 2 + A2 y + A3 (B) x = A1 y + A2
(C) x = A1 y + A2 y
2
(D) none of these
25. The solution of the differential equation
(x2 sin3 y – y2 cos x) dx + (x 3 cos y sin2 y – 2y sin x) dy = 0 is
(A) x3 sin3 y = 3y2 sin x + C (B) x3 sin3 y + 3y2 sin x = C
(C) x2 sin3 y + y3 sin x = C (D) 2x2 sin y + y2 sin x = C

Page # 27
DIFFERENTIAL EQUATION
dy
26. Solution of differential equation f(x) = f2 (x) + f(x) y + f ' (x) y is
dx
(A) y = f(x) + cex (B) y = – f(x) + cex
(C) y = – f(x) + cex f(x) (D) y = cf(x) + ex
27. Which one of the following is not homogeneous ?
xy x
(A) f(x, y)  (B) f(x, y)  x1/3 .y 2/3 tan1
x2  y2 y

(C) f(x, y)  x(ln x 2  y 2  ln y)  ye x/y

 2x 2  y 2  x  2y
(D) f(x, y)  x ln  ln(x  y)  y 2 tan
 x  3x  y

28. Solution of the equation  x  y  2dy  dx   3dx  5dy is

(A) 2 y  x  log  x  y  2   c (B) 2 x  y  log  y  x  2   c

(C) 2 y  x  log  x  y  2   c (D) 2x + y = log (y - x + 2) + c

 f y / x 
29. Solution of the equation xdy   y  x  is
 f '  y / x  

x y y y


(A) f    cy (B) f    cx (C) f    cxy (D) f    0
y x x x

dx x log x ey
30. Solution of the differential equation   if y (1) = 0, is
dy 1  log x 1  log x

(A) x x  e ye y (B) e y  x ey (C) x  ye y (D) y  e x


y

dy x 2  y 2  1
31. The solution of  , satisfying y 1  0, is given by
dx 2xy
(A) a hyperbola (B) a circle
(C) a ellipse (D) a parabola
x dy  y dx
32. Solution of the equation x dx  y d y  0
x2  y2

 c  x2  y 2   c  x2  y 2 
(A) y  x tan   (B) x  y tan  
 2   2 
 c  x2  y 2   c  x2  y2 
(C) y  x tan   (D) x  y tan  
 2   2 

dy
33. Solution of the equation  e x  y  e x  e y  is
dx
(A) e y  e x  1  ce ex (B) e y  e x  1  ceex

(C) e x  e y  1  ceey (D) e x  e y  1  ceey

Page # 28
DIFFERENTIAL EQUATION
34. If g(x) is a differential real valued function satisfying g”(x) – 3g’ (x) > 3  x  0 and g’
(0) = –1, then g(x) + x for x > 0 is
(A) an increasing function (B) a decreasing function
(C) a constant function (D) Data insufficient
2 2
dy  2  dy   dy 
35. Solution of the differential equation y   2
 x  2xy     0 is given by
 dx   dx   dx 
(A) y  2x  1 (B) y  x  2 (C) y  3x  2 (D) y  x  1
d2 y dy
36. If y = In (m cos–1 x) is a solution of the differential equation (1 – x2) 2
x  ke2y ,
dx dx
then k is equal to
(A) m2 (B) 2m2 (C) –m2 (D) –2m2
37. A curve is such that the area of the region bounded by the coordinate axes, the
curve and the ordinate of any point on it is equal to the cube of that ordinate. The
curve represents
(A) a pair of straight lines (B) a circle
(C) a parabola (D) an ellipse
d3 y dy
3
 13
38. If the function y = e4x + 2e–x is a solution of the differential equation dx dx  k
y
then the value of k is
(A) 4 (B) 6 (C) 9 (D) 12
39. A curve f(x) passes through the point P(1, 1). The normal to the curve at point P is a
(y – 1) + (x–1) = 0 If the slope of the tangent at any point on the curve is proportional
to the ordinate at that point, then the equation of the curve is
(A) y  eax  a (B) 1  eax (C) y  ea(x 1) (D) y  a  eax
40. If the length of the portion of the normal intercepted between a curve and the axis
varies as the square of the ordinate, then the curve is given by
(A) kx  1  k 2 x 2  ceky (B) ky  k 2 y 2  1  ce kx

(C) kx  k 2 x 2  1  ce xy (D) ky  k 2 y 2  1  ce xy
41. The equation of the curve in which the portion of the tangent included between the
coordinate axes is bisected at the point of contact, is
(A) a parabola (B) an ellipse (C) a circle (D) a hyperbola

x  y 
42. The solution of the differential equation 2 2
dy   2 2
 1 dx is
x y x y 

1 y
(A) y  x cot  c  x  (B) cos   x  c 
x

y2
(C) y  x tan  c  x  (D)  x tan  c  x 
x2

43. If   x  is a differentiable function, then the solution of the differential equation

dy  y  '  x     x   '  x  dx  0
 x 2
(A) y    x   1  ce   (B) y   x     x   c
 x
(C) ye  x     x  e   x   c (D) y     x   1  ce  

Page # 29
DIFFERENTIAL EQUATION

x dx  y dy y3
44. Solution of the differential equation  is given by
xdx  y dy x3

3/2
3 y x 3/2  y 3/2 y
(A) 2 log    log 3/2
 tan1   c 0
x x x

2 y x 3/2  y 3/2 y


(B) 3 log x  log 3/2
 tan 1  c  0
  x x

3/2
2 y xy 1  y 
(C) 3 log x  log  x   tan  3/2   c  0
    x 

3/2
1 3 3 1  y  c
(D) log  x  y   tan   
2 x 2

dy ax  3
45. If the solution of the differential equation  represents a circle, then the
dx 2y  f
value of 'a' is
(A) 2 (B) – 2 (C) 3 (D) – 4

d3 y d2 y dy
46. c1e 2x x
 c 2e  c 3e x
satisfies the differential equation 3
 a 2
b  cy  0 , then
dx dx dx
a 3  b3  c 3
is equal to
abc
1 1 1
(A) 1 (B) (C) – (D) –
2 2 4
dy 
47. The solution of the differential equation, 2x 2 y  tan(x 2 y 2 )  2xy 2 given y(1)  ,
dx 2
is
(A) sin x 2 y 2  ex 1 (B) sin(x 2 y 2 )  x
(C) cos x 2 y 2  x  0 (D) sin(x 2 y 2 )  e.e x
3
m1 x
48. The differential equation corresponding to y  c e 1 , where ci’ s are arbitary con-
i 1

st an t s an d m 1, m2, m3, are roots of m3 - 7m + 6 = 0 is

(A) y3 - 7y1 + 6y = 0 (B) y3 - 7y2 + 6y1 = 0

(C) y13  7y  6  0 (D) y 3  7y 2  y1  0

  1 
49. The solution of the differential equation  y 1    cos y  dx  (x  log x  x sin y)dy  0
  x 
is
(A) xy - y log x + x cos y = c (B) xy + y log x + x cos y = c
(C) xy - y log x - x log y = c (D) y log x + x cos y = c

Page # 30
DIFFERENTIAL EQUATION
x   x  
50. The solution of the differential equation xy log   dx   y 2  x 2 log    dy  0 is
y   y  

x2  x  x2 x2  x  x2
(A) log     log y  log c (B) log    log y  log c
2y 2  y  4y
2
2y 2  4y  4y
2

 x  x2 x2  x  x2
(C) log    2
 log y  log c  0 (D) log    log c  0
 y  4y 2y 2  y  4y
2

MULTIPLE CORRECT TYPE QUESTION


dy
1. The solution of  x  xe(n1)y is
dx
1  e(n1)y  1  x 2 x2 /2
(A) n  1 log  (n1)y   C (B) e(n1)y  Ce(n 1)y (n1) 1
 e  2

 e(n1)y  1 
(C) log  (n 1)y 
 x2  C (D) e(n1)y  Ce(n1)x2 /2 x  1
 (n  1)e 
2. The orthogonal trajectories of the family of coaxial circle x2 + y2 + 2 gx + C = 0,
where is a parameter are
(A) family of circles with centre on y-axis
(B) system of coaxial parabolas
(C) x 2  y 2  C' x  Cy  0 , where C’ is an arbitray constant
(D) system of coaxial circles with radical axis along x-axis
3. Let f(x) be a non-zero function, whose all successive derivatives exist and are non-
zero. If f(x), f’(x) are in G.P. and f(0) = f’(0) =1
(A) f(x)  0x  R (B) f '(x)  0x  R
(C) f ''(x)  1 (D) f(x)  lx  R
d2 y dy
4. Let y   A  Bx  e 3x
is a solution of the differential equation 2
m  ny  0,m,nI,
dx dx
then
(A) m = – 6 (B) n = – 6 (C) m = 9 (D) n = 9
2
 dy  dy
5. The solution of the equation    2y cot x  y 2 is
 dx  dx

C C
(A) y  0 (B) y 
1  cos x 1  cos x

C 1 C
(C) x  2sin1 (D) y  2 cos
2y 2y
6. The differential equation of the curve for which the initial ordinate of any tangent
is equal to the corresponding subnormal
(A) is linear in x (B) is homogenous of first degree
(C) has separable variables (D) is of second order
7. The curve for which the area of the triangle formed by the x-axis, the tangent line
at any point P and line OP is equal to a2 is
a2 a2 a2
(A) x  cy  (B) y  x  cx 2
(C) y  cx  (D) x  cy 
y x y

Page # 31
DIFFERENTIAL EQUATION
8. The curve C passes through (1, 1) in which the portion of tangent included between
the coordinate axes is bisected at point of contact
(A) The equation of curve C is xy = 1
(B) The equation of curve C is x2y = 1
 1
(C) The curve C passes through  2, 
 2

 1
(D) The curve C passes through  2, 
 4
9. Equation of a curve in whcih the subnormal is twice the square of ordinate is given
by
(A) log y = 2x + log c (B) y = ce2x
(C) log y = 2x – log c (D) y  ce x

d2 y
10. The differential equation + y + cot2 x = 0 must be satisfied by
dx 2

(A) y = 2 + c1 cos x + c 2 sin x

 x
(B) y = cos x . ln  tan  + 2
 2
 x
(C) y = 2 + c1 cos x + c2 sin x + cos x log  tan 
 2
(D) all the above
11. The solution of x 2 y12 + xy y1 – 6y2 = 0 are
(A) y = Cx 2 (B) x2 y = C
1
(C) ln y = C + log x (D) x3 y = C
2

dn y
12. If y = e–x cos x and yn + k n y = 0, where y n = and k n, n  N are constants.
dx n
(A) k4 = 4 (B) k 8 = –16 (C) k 12 = 20 (D) k 16 = –24

dy 1  y2
13. Solution of the differential equation + = 0 is
dx 1  x2
(A) tan–1 y + sin–1 x = c (B) tan–1 x + sin–1 y = c
1
(C) tan–1 y . sin–1 x = c (D) cot–1 + cos–1 1 x 2 = c
y
14. The solution of (x + y + 1) dy = dx are
(A) x + y + 2 = Ce y (B) x + y + 4= = C log y
(C) log (x + y + 2) = Cy (D) log (x + y + 2) = C + y
2
 dy 
15. The orthogonal trajectories of the system of curves   = a/x are
 dx 

2
(A) 9 a(y + c) 2 = 4x3 (B) y + C = x3/2
3 a

2
(C) y + C = x3/2 (D) none of these
3 a

Page # 32
DIFFERENTIAL EQUATION

2 dy 1 1
16. Solution of the differential equation x .cos  y sin  1 where y  1 as
dx x x
x   , is
1 1 x 1
(A) y  sin  cos (B) y 
x x 1
x sin
x
1 1 x 1
(C) y  cos  sin (D) y 
x x 1
x cos
x
dy sin2 x
17. A function y = f(x) satisfying the differential equation .sin x – y cos x + 0
dx x2
is such that , y =  as x   then :
/2

(A) Limit f(x)  1 (B)  f(x)dx is less than
x 0
0
2
/2

(C)  f(x)dx
0
is greater than unity (D) f(x) is an even function

dy f(xy)
18. If x yx , then f (xy) is equal to
dx f '(xy)
x2 y2 x2 xy
c
(A) ke 2 (B) ke 2
(C) ke 2 (D) ke 2

d  dx
19. The function f()   satisfies the differential equation
d 1  cos  cos x
0

df df
(A)  2f()cot   0 (B)  2f()cot   0
d d
(C) f()  1  cot2  (D) f()  1  cos ec2
20. A curve C has the property that if the tangent drawn at any point P on C meets
the axes at A and B then P is the mid point of AB. If the curve passes through
(1, 1) then
(A) the curve is a rectangular hyperbola
(B) the curve is a conjugate hyperbola of the hyperbola xy = –1
(C) the curve is a circle x 2 + y2 = 2
(D) the curve is a parabola y 2 = x

MATCH THE COLUMN TYPE QUESTIONS


1. Match the following
Column - I Column - II

xdy dy
(A) Solution of y – = y2 + is (p) xy2 = 2y5 + c
dx dx

dy
(B) Solution of (2x – 10y3) + y = 0 is (q) sec y = x + 1 + cex
dx

Page # 33
DIFFERENTIAL EQUATION
(C) Solution of sec y dy + tan y dx = dx is
2
(r) (x + 1) (1 – y) = cy

dy
(D) Solution of sin y = cos y (1 – x cos y) is (s) tan y = 1 + ce–x
dx
2. Match the following
Column - I Column - II

1
(A) xdy = y(dx + ydy), y(1) = 1 and y(x0) = –3, then x0 = (p)
4

dy
(B) If y(t) is solution of (t + 1) – ty = 1, (q) –15
dt
y (0) = –1, then y (1) =

1
(C) (x2 + y2) dy = xydx and y(1) = 1 and (r) –
2
y(x0) = e, then x0 =

dy 2y
(D) + = 0, y (1) = 1, then y(2) = (s) 3e
dt x

COMPREHENSION TYPE QUESTIONS


Comprehension - 1
Differential equations are solved by reducing them to the exact differential of
an expression in x & y i.e., they are reduced to the form d(f(x, y)) = 0

xdx  ydy ydx  xdy


e.g. : 2
x y 2 =
x2


1 2xdx  2ydy
2 x2  y 2
= –
xdy  ydx
x2
 d  x2  y 2  = – d  xy 
 2 2 y
 d  x y   = 0
 x

y
 solution is x2  y2 + = c.
x
Use the above method to answer the following question (3 to 5)
1. The general solution of (2x 3 – xy2) dx + (2y3 – x2y) dy = 0 is
(A) x4 + x2y2 – y4 = c (B) x4 – x2y2 + y4 = c
(C) x4 – x2y2 – y4 = c (D) x4 + x2y2 + y4 = c
xdy  y 
2. General solution of the differential equation x 2  y 2 + 1  2 2  dx = 0 is
 x y 
y x
(A) x + tan–1   = c (B) x + tan–1 y = c
x

y
(C) x – tan–1   = c (D) none of these
x

Page # 34
DIFFERENTIAL EQUATION
3. General solution of the differential equation e dx + (xe – 2y) dy = 0 is
y y

(A) xey – y2 = c (B) yex – x2 = c (C) yey + x = c (D) xey – 1 = cy2

Comprehension - 2
dn y dn 1 y
In order to solve the differential equation of the form a0 + a + ..... + any = 0,
dx n 1
dx n1
where a0, a1, a2 are constants.
We take the auxiliary equation a0Dn + a1 Dn–1 + ....+ an = 0
Find the roots of this equation and then solution of the given differential equation
will be as given in the following table.
Roots of the auxiliary equation Corresponding comple
mentary function

1. One real root a1 c1e1x

2. Two real and different roots a1 and a2 c1e1x  c 2e2 x

3. Two real and equal roots a1 and a1 (c1 + c2x) e 1x


1x
4. Three real and equal roots a1, a1, a1 (c1 + c2x + c3x2) e
5. One pair of imaginary roots a ± ib (c1 cos bx + c2 sin bx) eax
6. Two pair of equal imaginary roots a ± ib and a ± ib [(c1 + c2x) cos bx + (c1 +
c2x) sin bx] eax
Solution of the given differential equation will be y = sum of all the corresponding
parts of the complementary functions.

d2 y dy
4. Solve 2 – 2 + y = 0.
dx dx
(A) y = (c1 + c2x)ex (B) y = (c1ex + c2ex)
(C) y = (c1x)ex (D) none of these

d2 y
5. Solve + a2y = 0.
dx 2
(A) y = (c1 cos ax + c2 sin ax)eax (B) y = c1 cos ax + c2 sin ax
(C) y = c1 e ax
+ c2 e –ax
(D) none of these

d3 y d2 y dy
6. Solve – 6 2 + 11 – 6y = 0
dx 3
dx dx
(A) y = (c1 + c2 x + c3 x2)e x (B) y = x (c1 ex + c2 e2x + c3 e3x)
(C) y = c1 ex + c2 e2x + c3 e3x (D) none of these

Page # 35
DIFFERENTIAL EQUATION
INTEGER ANSWER TYPE QUESTION

1. A curve passing through the point (1,1) has the property that the perpendicular
distance of the origin from normal at any point p of the curve is equal to the dis-
tance of p from the x-axis is a circle with radius

 x3 c
2. The curves which intersect the hyperbola xy=4 at an angle is y   then
2 k 4
k
the value of is
3
3. The curve represented by the differential equation xdy – ydx = ydy intersects the y
axis at A(0, 1) and the line y = e at (a, (B) then find a + b

2
d 2 y  dy  x3
4. If soluton of y 2
    x is given by y 2   c1x  c 2 then the numerical quantity
dx  dx  
 should be equal to

5. If f(x) & g(x) are twice differentiable functions on [0, 2] satisfying


f "(x)  g "(x) f '(1)  2, g '(1)  4, f (2)  3, g(2)  9 then find g(1) – f(1)

6. If f:R  1  R and f is differentiable function satisfies f x  f  y   xf  y   


 y  f  x   yf  x   x, y R  1 then find the value of 2010 [1 + f(2009)]
7. Let y = f(x) be a curve passing through (e, ee), which satisfy the differential equation
e

(2ny + xy logex) dx – x logex dy = 0, x > 0, y > 0. If g(x) = lim


n
f(x) then  g(x)dx equals
1/e

to

Questions asked in previous AIEEE / JEE MAINS

n , then (1 + x2)
d2 y dy
1. If y = (x + 1 x2) 2 + x is [AIEEE 2002]
dx dx

(A) n2 y (B) –n2 y (C) – y (D) 2x2 y


2/3
 dy  d3 y
2. The order and degree of the differential equation 1  3  = 4 are
 dx  dx 3
[AIEEE 2002]

 2
(A) 1,  (B) (3, 1) (C) (3, 3) (D) (1, 2)
 3

d2 y
3. The solution of the equation = e–2x is [AIEEE 2002]
dx 2

e –2x e –2x 1 –2x 1 –2x


(A) (B) + cx + d (C) e + cx2 + d (D) e +c+d
4 4 4 4

Page # 36
DIFFERENTIAL EQUATION

4. The degree and order of the differential equation of the family of all parabolas
whose axis is x-axis, are respectively [AIEEE 2003]
(A) 2, 1 (B) 1, 2 (C) 3, 2 (D) 2, 3

1 y dy
5. The solution of the differential equation (1 + y2) + (x – e tan ) = 0, is
dx
–1 –1 –1
tan y 2tan y
(A) (x – 2) = k etan y
(C) 2x e = e +k [AIEEE 2003]
–1
–1 –1 tan y
(C) x etan y
= tan–1 y + k (D) x e2tan y
= e +k
6. The differential equation for the family of curves x2 + y2 – 2ay = 0, where a is an
arbitrary contant, is [AIEEE 2004]
(A) 2(x2 – y2) y ' = xy (B) 2(x2 + y2) y ' = xy
(C) (x2 – y2) y ' = 2xy (D) (x2 + y2) y ' = 2xy
7. The solution of the differential equation y dx + (x + x2y) dy = 0 is- [AIEEE 2004]

1 1 1
(A) – xy = c (B) – xy + log y = c (C) xy + log y = c (D) log y = cx

8. The differential equation representing the family of curves y2 = 2c(x + c ), where


c > 0, is a parameter, is of order and degree as follows- [AIEEE 2005]
(A) order 2, degree 2 (B) order 1, degree 3
(C) order 1, degree 1 (D) order 1, degree 2

dy
9. If x = y(log y – log x + 1), then the solution of the equation is [AIEEE 2005]
dx

x y y x


(A) log   = cy (B) log   = cx (C) x log   = cy (D) y log   = cx
y x x y
10. The differential equation whose solution is Ax 2 + By2 = 1, where A and B are
arbitary constant, is of [AIEEE 2006]
(A) first order and second degree (B) first order and first degree
(C) second order and first degree (D) second order and second degree
11. The differential equation of all circles passing through the origin and having their
centres on the x-axis is- [AIEEE 2007]
dy dy
(A) x2 = y2 + xy (B) x2 = y2 + 3xy
dx dx
dy dy
(C) y2 = x2 + 2xy (D) y2 = x2 – 2xy
dx dx
12. The normal to a curve at P(x, y) meets the x-axis at G. If the distance of G from the
origin is twice the abscissa of P, then the curve is a [AIEEE 2007]
(A) ellipse (B) parabola (C) circle (D) hyperbola

dy xy
13. The solution of the differential equation = satisfying the condition
dx x
y (1) = 1 is [AIEEE 2008]
(A) y = log x + x (B) y = x log x + x2 (C) y = xe(x – 1) (D) y = x log x + x

Page # 37
DIFFERENTIAL EQUATION
14. The differential equation which represents the family of curves y = c1 e c2 x , where c1
and c2 are arbitary constants is [AIEEE 2009]
(A) y ' = y2 (B) y ' ' = y ' y (C) y.y ' ' = y ' (D) y.y ' ' = (y ' )2

15. Solution of the differential equation cosx dy = y(sinx – y) dx, 0 < x < is
2
(A) y sec x = tan x + c (B) y tan x = sec x + c [AIEEE 2010]
(C) tanx = (sec x + c)y (D) secx = (tanx + c) y
dy
16. If  y  3  0 and y(0) = 2, then y (in 2) is equal to [AIEEE 2011]
dx
(A) 5 (B) 13 (C) –2 (D) 7
17. Let I be the purchase value of an equipment and v(t) be the value after it has been
used for t years. The value V(t) depreciates at a rate given by differential equation
dV(t)
 k(T  t) , where k > 0 is a constant and T is the total life in years of the
dt
equipment. Then the scrap value V(T) of the equipment is [AIEEE 2011]
kT 2 k(T  t)2 2 1
(A) I (B) I  (C) e -kT (D) T 
2 2 k
18. The population p(t) at time t of a certain mouse species satisfies the differential
dp(t)
equation  0.5p(t) – 450. If p(0) = 850, then the time at which the population
dt
becomes zero is [AIEEE 2012]
1
(A) 2 ln 18 (B) ln 9 ln18 (C) (D) ln 18
2
19. At present, a firm is manufacturing 2000 items. It is estimated that the rate of
change of production P w.r.t. additional number of workers x is given by
dP
 100  12 x . If the firm employs 25 more workers, then the new level of
dx
production of items is: [JEE MAINS - 2013]
(A) 3500 (B) 4500 (C) 2500 (D) 3000
20. Let the population of rabbits surviving at a time t be governed by the differential
dp  t  1
equation  p  t   200 . If p(0) = 100, then p(t) equals [JEE MAINS - 2014]
dt 2
(A) 400 – 300 et/2 (B) 300 – 200 e–t/2 (C) 600 – 500 et/2 (D) 400 – 300 e–t/2

dy
21. Let y(x) be the solution of the differential equation  x log x   y  2x log x,  x  1
dx
Then y(e) is equal to : [JEE MAINS - 2015]
(A) e (B) 0 (C) 2 (D) 2e
22. If a curve y = f(x) passes through the point (1, –1) and satisfies the differential
 1
equation, y(1 + xy) dx = x dy, then f    is equal to: [JEE MAINS - 2016]
 2

4 2 4 2
(A)  (B) (C) (D) 
5 5 5 5
Page # 38
DIFFERENTIAL EQUATION

Questions asked in previous AIEEE / JEE MAINS


1. A country has a food deficit of 10 %. Its population grows continuously at a rate
of 3 % per year. Its annual food production every year is 4 % more than that of
the last year. Assuming that the average food requirement per person remains
constant, prove that the country will become self-sufficient in food after ' n '
 n 10   n 9
years , where ' n ' is the smallest integer bigger than or equal to, .
 n (104
. )  0.03

[IIT-JEE-2000]
x
2. Let f(x), x  0, be a non-negative continuous function, and let F(x) =  f(t) dt , x  0.
0
If for some c > 0, f(x)  cF(x) for all x  0, then show that f(x) = 0 for all x  0
[IIT-JEE-2001]
3. A hemispherical tank of radius 2 meters is initially full of water & has an outlet
of 12 cm 2 cross-sectional area at the bottom. The outlet is opened at some
instant. The flow through the outlet is according to the law v(t) = 0.6 2gh(t) ,
where v(t) and h(t) are respectively the velocity of the flow through the outlet
and the height of water level above the outlet at time t & g is the acceleration
due to gravity. Find the time it takes to empty the tank. [IIT-JEE-2001]
dy
4. If y (t) is a solution of (1 + t) d t - t y = 1 and y (0) = - 1, then y (1) is equal to :
[IIT-JEE-2003]
1 1 1 1
(A) - (B) e + (C) e - (D)
2 2 2 2
5. An right circular cone of height H and radius R is pointed at bottom. It is filled
with a volatile liquid completely. If the rate of evaporation is directly proportional
to the surface area of the liquid in contact with air (constant of proportionality
k > 0) , find the time in which whole liquid evaporates. [IIT-JEE-2003]
dP(x)
6. If P(1) = 0 and > P(x) for all x  1 then prove that P(x) > 0 for all x > 1.
dx
[IIT-JEE-2003]
 2  sin x  dy  
7. If y = y(x) and  y  1  = – cosx ; y(0) = 1, then y   is equal to
  dx 2
[IIT-JEE-2004]
(A) 1/3 (B) –2/3 (C) 2/3 (D) –1/3
2
 x  1   y  3 
8. A curve passes through (2, 0) and slope at point P(x, y) is . Find
 x  1
equation of curve and area between curve and x-axis in 4th quadrant.
[IIT-JEE-2004]
9. 2 2
The solution y = y(x) of the differential equation (x + y ) dy = xy dx statisfies the
conditions y(1) = 1 and y(x0) = e, then value of x0 is – [IIT-JEE-2005]

(A) 3e (B) 2(e2  1) (C) 2(e2  1) (D) (e2  1)/ 2

Page # 39
DIFFERENTIAL EQUATION
10. Suppose y = y(x) satisfies the differential equation ydx + y2dy = xdy. If y(x) > 0 " x  R
and y(1) = 1, then y(–3) equals [IIT-JEE-2005]
(A) 1 (B) 2 (C) 3 (D) 5
11. If length of tangent at any point on the curve y = f(x) intercepted between the point
and the x-axis is of length 1. Find the equation of the curve. [IIT-JEE-2005]
12. A tangent drawn to the curve y = f(x) at P(x, y) cuts the x-axis and y-axis at A and B
respectively such that BP : AP = 3 : 1, given that f(1) = 1, then [IIT-JEE-2006]
dy
(A) equation of curve is x – 3y = 0 (B) normal at (1, 1) is 3y – x = 2
dx

dy
(C) curve passes through (2, 1/8) (D) equation of curve is x + 3y = 0
dx

t 2f(x)  x 2f(t)
13. Let f(x) be differentiable on the interval (0,  ) such that f(1) = 1 and lim =1
tx
tx
for each x > 0. Then f(x) is [IIT-JEE-2007]

1 2x 2 1 4x 2 1 2 1
(A) + (B) + (C) + 2 (D)
3x 3 3x 3 x x x

dy 1  y2
14. The differential equation = determines a family of circles with
dx y
(A) variable radii and a fixed centre at (0, 1) [IIT-JEE-2007]
(B) variable radii and a fixed centre at (0, –1)
(C) fixed radius 1 and variable centres along the x-axis
(D) fixed radius 1 and variable centres along the y-axis

15. Let a solution y = y(x) of the differential equation x x 2  1 dy – y y 2  1 dx = 0


2
satisfy y(2) = [IIT-JEE-2008]
3
 1 
Statement -1 : y(x) = sec  sec x  
 6
1 2 3 1
Statement-2 : y(x) is given by y = – 1
x x2
(A) Statement-1 is True, statement-2 is True ; Statement-2 is a correct explanation
for statement-1
(B) Statement-1 is True, statement-2 is True ; Statement-2 is NOT a correct
explanation for statement-1
(C) Statement-1 is True, statement-2 is False
(D) Statement-1 is False, statement-2 is True
x

16. Let f be a non-negative function defined on the interval [0, 1]. If  1  (f (t))2 dt
0
x

=  f(t) dt, 0  x  1 and f(0) = 0, then [IIT-JEE-2009]


0

1 1 1 1 1 1 1 1


(A) f   < and f   > (B) f   > and f   >
2 2 3 3 2 2 3 3

Page # 40
DIFFERENTIAL EQUATION

1 1 1 1 1 1 1 1


(C) f   < and f   < (D) f   > and f   <
2 2 3 3 2 2 3 3
17. Match the statements/expressions in Column - I with the open intervals in Column - II
[IIT-JEE-2009]
Column - I Column - II
  
(A) Interval contained in the domain of definition of (p)  – , 
 2 2
non-zero solutions of the differential
equation (x – 3)2 y ' + y = 0
 
(B) Interval containing the value of the integral (q)  0, 
 2
5

(x –1)(x – 2)(x – 3)(x – 4)(x – 5)dx


1

(C) Interval in which at least one of the points of local (r)   5 


 8, 4 
 
maximum of cos2x + sinx lies
 
(D) Interval in which tan–1 (sinx + cosx) is increasing (s)  0, 
 8
(t) (–  ,  )
18. Match the statements/expressions given in Column - I with the values given in
Column - II [IIT-JEE-2009]
Column - I Column - II

(A) The number of solutions of the equation xesinx – cos x = 0 (p) 1

 
in the interval  0, 
 2
(B) Value(s) of k for which the planes kx + 4y + z = 0, (q) 2
4x + ky + 2z = 0 and 2x + 2y + z=0 intersect in a straight line
(C) Value(s) of k for which |x – 1| + |x – 2| + |x + 1| + |x + 2| = 4k (r) 3
has integer solution(s)
(D) If y ' = y + 1 and y(0) = 1, then value(s) of y (ln 2) (s) 4
(t) 5
19. Let f be a real-valued differentiable function on R (the set of all real numbers) such
that f(1) = 1. If the y-intercept of the tangent at any point P(x, y) on the curve y = f(x)
is equal to the cube of the abscissa of P, then the value of f(–3) is equal to
[IIT-JEE 2010]
20. If y(x) satisfies the differential equation y ' - ytanx = 2x secx and y(0) = 0, then
[IIT-JEE 2012]

  
2
 2  2 
  4 22
(A) y   (B) y    (C) y    (D) y    
 4 8 2  4  18 3 9 3 3 3 3

Page # 41
DIFFERENTIAL EQUATION

 
21. A curve passes through the point 1,  . Let the slope of the curve at each point
 6
y y
(x, y) be  sec   .x  0 . Then the equation of the curve is [JEE Advanced 2013]
x x

y 1 y
(A) sin    log x  (B) cos ec    log x  2
x 2 x

 2y   2y  1
(C) sec    log x  2 (D) cos    log x 
 x   x  2

dy xy x4  2x
22. The function y = f (x) is the solution of the differential equation  2 
dx x  1 1  x2

3
2
in (-1, 1) satisfying f(0) = 0. Then  f(x)dx is [JEE ADVANCED 2014]
3

2

 3  3  3  3
(A)  (B)  (C)  (D) 
3 2 3 4 6 4 6 2

dy
23. A solution curve of the differential equation (x2 + xy + 4x + 2y + 4) – y2 = 0, x > 0,
dx
passes through the point (1, 3). Then the solution curve [JEE ADVANCED 2015
(A) intersects y = x + 2 exactly at one point.
(B) intersects y = x + 2 exactly at two points
(C) intersects y = (x + 2)2
(D) does NOT intersect y = (x + 3)2

Page # 42
DIFFERENTIAL EQUATION

ANSWER KEY
(DRILL-I)

1. (a) order = 2, degree = 3 (b) order = 2, degree = 3


(c) order = 1, degree = 1 (d) order = 2, degree = not defined
(e) order = 2, degree = not defined (f) order = 2, degree = not defined

1 1  x2 1 x2
2. 1  y2  1  x2  log C 3. y  tan 1
y  log x  C
2 1  x2 1 2
4. (i) tan (x + y) – sec (x + y) = x + C; (ii) – cot (x + y)+ cosec (x+y)= x + c;

1 1  4x  y  1 
(iii) 2 tan  2
xC
 

x y
5. ( x  y )  a tan 1  xC 6. – cosec (x + y) + cot (x + y) + y + C
 a 

x y
7. log 1  tan  xC
 2 

3 x  2y  2  2
8. 2 y  2 log | x 2  4 xy  4 y 2  4x  8 y  2 |  log C
2 x  2y  2  2

ba 2 x2
9. y  log | ( x  y )  ab | x  C 10. log | sin( y  x) | C
 2  2
11. (1 + log (x + y)) – log | 1 + log (x + y) | = x + C
1
12. sin x (1 + ey) = C 13. y  log | ( x  y )  2 | x  C
2
2

1 2a 3
14. (3x  4 y  5)  tan 1 (3x  4 y  5)  x  C
3 3 3 2a

dy dy
15. y – x + log | x + y | + C 16. (i) ( x 2  y 2 )  2 xy ; (ii) y 2  x 2  2 xy
dx dx

3 2
  dy  2  d2y   dy  2   dy 
2
2 
17. 1      r 2  2  18. ( x  y ) 1      x  y 
  dx    dx    dx    dx 
   

DRILL - II

1. x2(y2 + 2xy) = C 2. x2 – y2 = (x2 + y2)2 C2


3. xy cos (y/x) = C 4. xy sin (y/x) = p/2

( y  1) 2
5. x  ye x/ y
c 6.  log | x  1 | C
2( x  1) 2

 y2
7. [( x  1) 2  ( y  2) 2 ] tan 1    2( x  1)( y  2)  log C {x  1}
 x 1 

Page # 43
DIFFERENTIAL EQUATION

x2  x  x2
8. log2

y  4 y 2
  log y  log C 9. y2 – x2 + xy + 4y – 3x = C
2y 

10.
1 {( y 1) / x }
C x 2  ( y  1) 2  e tan

1 2( y  3)  (5  21)( x  2)  C 
11.  log  log  
 ( x  2) 2 
21 2( y  3)  (5  21)( x  2)

12. (cos y – sin x – 1)2 (sin x + cos y – 1)5 = C


13. (x + 3)3 – (y – 3)3 = C(x – y + 6)4
14. log (4x + 8y + 5) = 4x – 8y + C1, where C1 = 4C

d2x
16. 0
dy 2

DRILL - III
1. y ex = e x cos x  C 2. xy = x sin x  C

x y4
3. y
 2y C 4. xy  C
4

x
5.  y2  C 8. 1  log x  y  Cxy  1
y

1 2 2 1 1 
9. y 2
e x  e x  x  1  C
2
10. sec y.sec x  2  x  2 sin 2 x   C
 

2 1 2 2
11. e x tan y 
2

x 1 ex  C  12. y3  ax  C.x 1  x 2

1 1 1 2 1 1 
13. . 
log y x 2 x 2
C y 2
 2
14. e  2 cos y  sin y  2   c
 x 

2  2 tan( x  y )  1 
15. y (2 x )  2sin x  x or y  2sin x 16. tan 1    xc
3  3 
DRILL - IV
1 4
1. xy  x C 2. log(x2 +y2) = 2ay + C
2

1  y y 1
3. 
xy
 log   C 4.   x  cos y  C  0
x x x

2 2
5. 2x  y 2  (x  y 2 )3/ 2  C 6. ex/y = – cos y + C
3

 y x y
7. log x  cos   log C 8. x2  y2  1  x2  y2  C
x x y

2
x
9. x+ye =C 10.  cot( x  y )   y   C
2 2
x/y
 

 y x2
11. x 2  y 2  2 tan 1    C 3 2
12. x y  y  C
x

Page # 44
DIFFERENTIAL EQUATION
3 7
13. tan 2 x  7 tan x  cot 2 y  5 cot y  4 tan x cot y  C
2 2
2
 x  ( xy) 2 1x x3 x3
14. sin 
y  2
C 15.     (1  log x )
2 y 3

9
C

x3 y3  1  x  1 
16.   2 x 2 y  2 xy 2  C 17.   xy   ln | x |  ln | y | c or ln y  c   xy 
3 3    

18. ln( x 2  y 2 )  2 m tan 1 ( y / x )  c 19. x 2 ln y  y 3  c

x2   y 
20.  cot( x 2  y 2 ) 
y 2
c 21. x 2

 y 2  a sin  tan 1    c 
 x 
22. 2x2 + y2 = 2c 23. x2 – y2 = k

DRILL - V
(Word Problems)

y
2. sec    log | x |  2 3. x = Cy or xy = C
x

1  5 tan 4 x  
4. y   tan 1    5 x 5. xy = 2
3  4  3 tan 4 x  

6. 2y = –  x3 + 2Cx 7. y  y 2  C2   C e x / c
9. x2 + y2 = k 2 10. y2 + x2 = 2x
11. x + y = 2 and xy = 1 12. y2x = k2(1 – y2)

a2 2 2
13. y  Cx 14. f(x) =  x , g(x) = x +
2x x x
ln 2
15. t = ln(4 / 3) 16. 40 minutes

17. 48 years 18. T=H/k

SINGLE CORRECT QUESTIONS


1.A 2.D 3.A 4.D 5.A 6.C 7.B 8.B
9.A 10.B 11.C 12.A 13.B 14.C 15.B 16.A
17.B 18.C 19.B 20.C 21.A 22.A 23.D 24.A
25.A 26.C 27.D 28.A 29.B 30.A 31.A 32.C
33.A 34.A 35.D 36.C 37.C 38.D 39.C 40.B
41.D 42.C 43.A 44.D 45.B 46.D 47.A 48.A
49.B 50.A
MULTIPLE CORRECT TYPE QUESTION
1.AB 2.AB 3.ABC 4.AD 5.ABCD 6.AB 7. AD 8.AC
9.ABCD 10.BC 11.ACD 12. AB 13. AD 14. AD 15. ABC 16.A
17.ABD 18.AC 19.AC 20.AB

Page # 45
DIFFERENTIAL EQUATION

MATCH THE COLUMN TYPE QUESTIONS


1. (A) – (r); (B) – (p); (C) – (s); (D) – (q)
2. (A) – (q), (B) – (r), (C) – (s), (D) – (p)
COMPREHENSION TYPE QUESTIONS
1. B 2.A 3. A 4. A 5. B 6. C
INTEGER ANSWER TYPE QUESTION
1. (1) 2. (4) 3. (0) 4. (3)
5. (4) 6. (1) 7. (0)

Questions asked in previous AIEEE / JEE MAINS

1. A 2.C 3.B 4.B 5.B 6.C 7.B 8.B


9. B 10. C 11.C 12.A 13.D 14.D 15.D 16.D
17.A 18.A 19.A 20.A 21.C 22.C

PREVIOUS YEAR QUESTIONS (IIT-JEE)

14  10 5
3. 4. A 5. t = H/k 7. A 8. 4 / 3 9. A 10. C
27 g

1 1 y2
11. log + 1 y 2 = ± x + c 12. BCD 13. A 14. C
y

15. C 16.C
17. (A)  (p, q, s), (B)  (p, t), (C)  (p, q, r, t), (D)  (s)
18. (A)  (p), (B)  (q, s), (C)  (q, r, s, t), (D)  (r)
19. (9) 20. A, D 21. A 22. B
23. A,C

Page # 46
PROBABILITY 2
UNIT

 Theory

 Drill Exercises

 Solved Examples

 Exercises

 Single Correct Type Questions

 Multiple Correct Type Questions

 Matrix Match Type Questions.

 Comprehension Type Questions

 Integer Type Questions

 Previous Years Questions

 Answer Key
PROBABILITY
Terms and Definitions used in probability :
Random Experiment
An experiment is called random if
(i) all the outcomes of the experiment are known in advance
(ii) the exact outcome of any specific performance of the experiment is unpredicatable
i.e. not known in advance.
For Example drawing a card from a well shuffled pack of 52 playing cards is a
random experiment.
Sample Space
A set whose elements represent all possible outcomes of a random experiment is
called the sample space and is usually represented by ‘S’.
Consider the experiment of tossing a die. If we are interested in the number on the
top face, then sample space would be S1 = {1, 2, 3, 4, 5, 6}. If we are interested only
in whether the number is even or odd, then sample space is S2 = {even, odd}. Clearly
more than one sample space can be used to describe the outcomes of an experiment.
In this case ‘S1’ provides more information than ‘S2’. If we know which element in S1
occurs, we can tell which outcome in S2 occurs; however, knowledge of what happens
in S2 in no way helps us to know which element in S1 occurs.
In general it is desirable to use a sample space that gives the maximum information
concerning the outcomes of the experiment.
Sample Point
Each element of the sample space is called a sample point.
Event
An event is a subset of the sample space. When a die is rolled, sample space is S = {1,
2, 3, 4, 5, 6}. Let A = {1, 3, 5}, B = {2, 4, 6}, C = {1, 2, 3, 4}. Here A is the event of
occurrence of an odd number, B is the event of occurrence of an even number and C
is the event of occurrence of a number less than 5.
Simple Event
An event is called a simple event, if it is a singleton subset of the sample space S.
Compound Event
A subset of the sample space S which contains more than one element is called a
compound event
Equally likely Events
A set of events is said to be equally likely if taking into consideration all the relevant
factors there is no reason to expect one of them in preference to the others. For
example when a fair coin is tossed, the occurrence of a tail or a head are equally
likely.
Mutually Exclusive Events
A set of events is said to be mutually exclusive if the occurrence of one of them
precludes the occurrence of any of the remaining events. For example, when we
throw a pair of dice, the events “ a sum of 5 occurs”, “a sum of 7 occurs” and “a sum
of 9 occurs” are mutually exclusive. In set theoretic notation, events A1, A2, . . . , Am
are mutually exclusive if Ai  A j   for i  j and 1  i, j  m .
Exhaustive Events
A system of events is said to be exhaustive if on each performance of the experiment
at least one of the events of the system is must to occur. In set theoretic notation,
m
events A1, A2, . . . , Am are exhausitve if  Ai  S . For example on throwing of a die,
i 1
the events {1, 2}, {2, 3, 4}, {5} and {4, 5, 6} form an exhaustive system of events.

Page # 48
PROBABILITY

Probability with Discrete sample space containing equally likely points


If there are n exhausitve mutually exclusive and equally likely out comes of an
experiment and m of them are favourable to an event A, then the probability of the
m
happening of A is equal to and it is denoted by P(A). Clearly P(A) is a non-negative
n
number not greater than unity. So 0  P(A)  1 .
If probability of happening of an event A is 1, then A is called certain event and if
probability of happening of event A is zero, then A is called impossible event.
Axiomatic Definition of probability

If S is simple space of a random experiment, A real valued function from p : P  s   R


(where P(s) is power set of S) is called probability function if ‘p’ satsifies the following
three axioms

(i) Axiom of positivity : If E  S, P  E  0


(ii) Axiom of Certainity : P(S) = 1
(iii) Axiom of union : If E 1 , E 2 are mutually exclusive events then
P  E 1  E2   P  E1   P  E2 
Ex.1 Six boys and six girls sit in a row randomly, find the probability that all the 6 girls sit
together.
Sol. Let S be the sample space and E the event that all the 6 girls sit together .
Then n(S) = Total number
of ways of seating 6 boys and 6 girls in a row = 12! and n(E) = number of ways of
seating 6 boys and
6 girls in a row so that all the 6 girls sit together = 7!6!

n  E 6!7! 1
 P(E) = n  S  12!  132 .

Ex.2 A bag contains 5 red and 4 green balls. Four balls are drawn at random then
find the probability that two balls are of red and two balls are of green colour.
Sol. n(s) = the total number of ways of drawing 4 balls out of total 9 balls : 9C4.
If A1 = the event of drawing 2 red balls out of 5 red balls then n(A 1) = 5C2
A2 = the event of drawing 2 green balls out of 4 green balls then n(A 2) = 4C2
 n(A) = n(A1) . n(A2) = 5C2 ×4C2

5443
5 4
n(A) C2  C2 2 2 10
 P(A) = n(s) = 9
C4
= 9876 =
21
432
Ex.3 If three cards are drawn from a pack of 52 cards, what is the chance that all
will be queen?
Sol. If the sample space be s then n(s) = the total number of ways of drawing 3 out
of 52 cards = 52C3
Now, if A = the event of drawing three queens then n(A) = 4C3

Page # 49
PROBABILITY

n(A) 4 C3 4 1
 P(E) = n(s) = 52 C = 52  51  50 =
3 5525
32
Ex.4 5 cards are drawn from a pack of 52 cards what is the probability that these
5 will contain just one king?
Sol. The total ways of drawing 5 out of 52 cards i.e., the number of elements in
sample space S . n(s) = 52C5.
Out of 52 cards, there are 4 kings and 48 other cards. Out of 4 kings the
4
total number of ways of drawing 1 king = C1 . So if E1 = the event of having
one King n(E1) = 4C1.
Remaining 4 can be drawn out of 48 in 48C4 ways.
So, if E2= the event of having any 4 from the remaining cards.
n(E2) = 48C4
Let E = the event of drawing, 1 king and 4 other cards.
then n(E) = n (E1) . n(E2) = 4C1 × 48C4

n(E) 4 C1  48 C4
 P(E1) = n(s) = 52
C5

4! 48! 5! 47! 3243


= 1! 3! × 4! 44! × =
52! 10829

DRILL - I
Axiomatic Definition of Probability
1. The probability that a card drawn from a pack of 52 cards will be a diamond or king is

4 4 1 2
(A) (B) (C) (D)
52 13 52 13
2. Three mangoes and three apples are in box. If two fruits are chosen at random, the
probability that one is a mango and the other is an apple is

2 3 1 1
(A) (B) (C) (D) .
3 5 3 5
3. From 4 childreen, 2 women and 4 men, 4 are selected. The probability that there are
exactly 2 children among the selected is

11 9 10 8
(A) (B) (C) (D)
21 21 21 25

4. A bag contains 3 red and 3 white balls. Two balls are drawn one by one. The probability
that they are of different colours is
3 2 3 1
(A) (B) (C) (D) .
10 5 5 5
5. A bag contains 5 brown and 4 white socks. A man pulls out 2 sockes. The probability
that they are of the same colour is
5 1 5 4
(A) (B) (C) (D)
108 6 18 9

Page # 50
PROBABILITY
6. Two dice are thrown together. The probability of getting the sum of digits as a multiple
of 4 is
1 1 1 5
(A) (B) (C) (D)
9 3 4 9
7. If a person throws 3 dice, the probability of getting sum of digit exactly 15 is-
5 5 5 1
(A) (B) (C) (D) .
72 108 36 72
8. If the letters of INTERMEDIATE are arranged, then the probability no two E’s occur
together is-
6 5 2 3
(A) (B) (C) (D)
11 11 11 11
9. One number is selected at random from first two hundred positive integers. The
probability that it is divisible by 6 or 8 is
1 2 3 1
(A) (B) (C) (D)
3 3 4 4
10. Four persons are selected at random out of 3 men, 2 women and 4 children. The
probability that there are exactly 2 children in the selection is
11 9 10 12
(A) (B) (C) (D)
21 21 21 21
11. A bag contains 5 white and 3 black balls. Two balls are drawn at random. The probability
that one ball is white and other is black will be
15 2 8 1
(A) (B) (C) (D)
28 7 28 7
12. Three letters are written to different persons, and addresses on three envelopes are
also written. Without looking at the addresses, the probability that the letters go into
right envelopes is

1 1 1 1
(A) (B) (C) (D)
27 6 9 3
13. A natural number is chosen at random from the first one hundred natural numbers.

The probability that


 x  20  x  40   0 is
x  30
1 3 3 7
(A) (B) (C) (D)
50 50 25 25
14. A is a set containing n elements. A subset P1 of A is chosen at random. The set A is
reconstructed by replacing the elements of P1. A subset P2 is again chosen at random.
The probability that P1  P2 contains exactly one element, is

3n 3n 3 3
(A) n (B) n (C) (D) .
4 4 4 4n
15. A drawn two cards at random from a pack of 52 cards. After returning them to the
pack and shuffling it, B draws two cards at random. The probability that there is
exactly one common card, is
5 50 25 25
(A) (B) (C) (D)
546 663 663 273

Page # 51
PROBABILITY
16. x1, x2, x3, . . . , x50 are fifty real numbers such that xr < xr + 1 for r = 1, 2, 3, . . ., 49. Five
numbers out of these are picked up at random. The probability that the five numbers
have x20 as the middle number is
20
C2 30 C2 30
C2  19 C2 19
C2 31 C3 0
C2 30 C2
(A) 50 (B) 50 (C) 50 (D) 49 .
C5 C5 C5 C5

17. If the integers m and n are chosen at random from 1 to 100, then the probability that
a number of the form 7n + 7m is divisible by 5 equals
1 1 1 1
(A) (B) (C) (D)
4 2 8 3
18. Two numbers are selected at random from 40 consecutive natural numbers. The
probability that the sum of the selected numbers is odd will be
14 20 1 19
(A) (B) (C) (D)
29 39 2 39
19. If all letters of the word MISSISSIPPI are rearranged then the probability that all S
come together will be
1 4 8 2
(A) (B) (C) (D)
165 165 165 165
20. If the letter of the word SUCCESS are arranged, then the probability that similar
letters occurs together is
4 3 1 2
(A) (B) (C) (D)
35 35 35 35
21. If two persons p1 and p2 throw a dice, the probability that p1 throws a number higher
than p2 throws is
1 15 1 11
(A) (B) (C) (D) .
2 36 36 36
22. If the letters of the word ATTEMPT are written down at random, the chance that all
Ts are consecutive is-
1 6 1 41
(A) (B) (C) (D)
42 7 7 42
23. 4 gentlemen and 4 ladies take seats at random round a table. The probability that
they are sitting alternately is-
4 1 2 1
(A) (B) (C) (D)
35 70 35 35
24. From a group of 10 persons consisting of 5 lawyers, 3 doctors and 2 engineers, four
persons are selected at random. The probability that the selection contains at least
one of each category is-
1 1 2 1
(A) (B) (C) (D)
2 3 3 4
25. A and B draw two cards each, one after another, from a pack of well shuffled pack of
52 cards. The probability that all the four cards drawn are of the same suit is-
44 11 13  24 11
(A) (B) (C) (D)
85  49 85  49 17  25  49 17  49
26. Three numbers are chosen at random without replacement from the set
A = {x | 1  x  10, x  N}. The probability that the minimum of the chosen numbers is
3 and maximum is 7, is
1 1 1 39
(A) (B) (C) (D)
12 15 40 40

Page # 52
PROBABILITY

Odds in Favour and Odds Against an Event


As a result of an experiment, if p of the outcomes are favourable to an event E and q
of the outcomes are against it, then we say that odds are p to q in favour of E or odds
are q to p against E.
number of favourable cases p 4
 Odds in favour of an event E = number of unfavourable cases  q 52  4

number of unfavourable cases q


and odds against an event E = 
number of favourable cases p

If odds in favour of an event E are p : q then the probability of the occurrence of that
p
event is p  q

q
Similarly the probability of the non-occurrence of that event is p  q

Ex.5 If a card is drawn from a pack of 52 cards then find the odds in favour of the
event that it is a card of a queen.
Sol. Here n = 52, m = 4

4 4
Odds in favour of the event = = = 1 : 12
52  4 48
Ex.6 What are the odds in favour of throwing at least 8 in a single throw with two dice?
Sol. Let S be the sample space and E be the event of occurrence of a total of at least 8,
then
n(S) = 6  6  36 and n(E) = 15
15
 P(E) =
36
15 21
 P  E  1  P(E)  1  36  36

 
Odds infavour of E = P(E) : P E = 5 : 7
DRILL - II
1. The odd against throwing 10 with two dice in a throw are-
(A) 9 : 1 (B) 10 : 1 (C) 11 : 1 (D) 12 : 1
2. From a pack of well shuffled cards, one card is drawn randomly. A gambler bets
that it is either a diamond or a king. The odds in favour of his winning the bet
is-
(A) 4 : 7 (B) 4 : 9 (C) 9 : 4 (D) None of these
3. A bag contains 4 red and 4 white balls. Three balls are drawn at random. The
odd against these balls being all white are-
(A) 1 :13 (B) 13 : 1 (C) 12 : 1 (D) 14 : 1
4. One of the two events must occur. If the chance of one is 2/3 of the other, then
odds in favour of the other are-
(A) 2 : 3 (B) 1 : 3 (C) 3 : 1 (D) 3 : 2
5. If odds in favour of A is m : n then P(A) equals-
(A) m/n (B) m – n
(C) m / (m + n) (D) m / (m – n)
Page # 53
PROBABILITY
6. A dice is thrown then the odds against of getting the digit 6 is-
(A) 5 : 1 (B) 4 : 1 (C) 6 : 1 (D) None of these
7. If one card is drawn from a pack of card then the odds in favour of getting Ace
is-
(A) 1 : 3 (B) 3 : 1 (C) 1 : 2 (D) 1 : 12
8. A bag contains 3 black and 2 white balls. What are the odds in favour of drawing
a white ball?
(A) 3 : 2 (B) 2 : 5 (C) 2 : 3 (D) 3 : 5

Set Theoretic Principles


If ‘A’ and ‘B’ be any two events of the sample space, then A B would stand for
occurrence of atleast one of them and AB stands for simultaneous occurrence of A
and B. A  or A  stands for non–occurrence of A. A  B stands for non–occurrence of
A or B. A  B denotes non-occurrence of both A & B simultaneously
If A and B are any two events, then P(AB) = P(A) + P(B) – P (AB). If A and B are
mutually exclusive, P(AB) = P(A) + P(B).

 
Hence P A = 1 – P(A)

P(A B ) = P(A) – P(AB)


Now P(exactly one of A and B occurs)

= P  A  B   P  A  B  = P(A) + P(B)  2P(A  B) = P(A  B)  P(A  B)


Some Theorems

(i) If A  B , then (i) P  A   P  B  and (ii) P(B – A) = P(B) – P(A)

 n  n
(ii) P 
  Ai  
  P  Ai  and equality holds if and only if events Ai, i = 1, 2, . . . , n are
 i1  i 1
exclusive.

 n 
(iii)   Ai   1 and equality holds if and only if events Ai, i = 1, 2, . . . n are exhaustive.
P
 i 1 

n
(iv)  P  Ai  = 1 if events Ai, i = 1, 2, . . ., n are exclusive and exhaustive.
i 1

 n  n
  i    i 
Note : If events A1, A2, . . . , An are exclusive, then P  A  P A . This is called
 i 1  i 1
the rule of sum.

Some Useful Formulas


(i) For two events A and B
(a) P(at least one out of them) = P(A) + P(B) – P( A  B)
(b) P (exactly one out of them) = P(A) + P(B) – 2 P( A  B)
(ii) For three events A, B and C
(a) P(at least one out of them) =
Page # 54
PROBABILITY
P  A   P  B  P  C   P  A  B   P  B  C   P  C  A   P  A  B  C 
(b) P (at least two out of them) =
P( B  C)  P(C  A )  P( A  B)  2P( A  B  C)
(c) P (exactly two out of them) =
P( B  C)  P(C  A )  P( A  B)  3P( A  B  C)
(d) P (exactly one out of them) =
P(A )  P( B)  P(C)  2P( B  C)  2P(C  A )  2P( A  B)  3P( A  B  C)
Ex.7 From the set S = {1, 2, 3, . . . 3n}, three numbers are chosen at random. Find the
probability that the sum of the chosen numbers is divisible by 3.
Sol. Let Sk  S, having those numbers, which leave a remainder of k, k = 0, 1, 2, when
divided by 3. Obviously n(Sk) = n.
Let Ek be the event that the numbers are chosen from Sk, k = 0, 1, 2. Let E3 be the
event that the chosen numbers are from the sets S0, S1 and S2 (one number from
each set). Let E be event that the sum of the chosen numbers is divisible by 3. Now
n
C3
P(E0) = P(E1) = P(E3) = 3n .
C3
nnn
P(E3) = 3n .
C3
The required probability = P(E) = P  E0  E1  E2  E3 

3.n C3  n3
= P(E0) + P(E1) + P(E2) + P(E3) (using rule of sum ) = 3n .
C3
Ex.8 One number is selected from first 20 positive integers. What is the probability that
it is divisible by 3 or 4.
(A) 1/5 (B) 1/2 (C) 3/16 (D) 1/9
Sol. Let A = event that selected number is divisible by 3
B = event that selected number is divisible by 4
Here the events are not mutually exclusive.
6 5
then P (A) = , P (B) = ,
20 20
1
P (AB) =
20
 P (A + B) = P (A) + P (B) – P (AB)
6 5 1 10 1
= + – = = Ans. [B]
20 20 20 20 2
Ex.9 If two cards are drawn from a pack of cards then the probability of getting at least
one Ace is -
(A) 1/5 (B) 33/221 (C) 3/16 (D) 1/9
Sol. The Total number of exhaustive cases of drawing two cards = 52C2
Now, to get at least one Ace out of two drawn cards , one card of Ace and second
card of others or both cards can be of Ace.

Page # 55
PROBABILITY
If these events are denoted by A and B respectively then
P(A) = (4C1 × 48C1)/ 52C2 = 32 /221,
P(B) = 4C2 / 52C2 = 1/221
 P (A + B) = P(A) + P (B)
 P (A + B) = 32/221 + 1/221 = 33/221

DRILL - III
1. Two dice are thrown together. The probability that atleast one will show its digit
greater than 3 is
1 3 1 1
(A) (B) (C) (D)
4 4 2 8
2. The probability that an anti aircraft gun can hit an enemy plane at the first, second
and third shot are 0.6, 0.7 and 0.1 respectively. The probability that the gun hits the
plane is
(A) 0.108 (B) 0.892 (C) 0.14 (D) 0.91
3. The probability that atleast one of the events A and B happens is 0.6. If probability of
their simultaneous happening is 0.2, then P  A   P B  is
(A) 0.4 (B) 0.8 (C) 1.2 (D) 1.4
4. A number is chosen at random among the first 120 natural numbers. The probability
of the number chosen being a multiple of 5 or 15 is

1 1 1 1
(A) (B) (C) (D)
5 8 6 4

5 1 3
5. If A and B are two events such that P(A  B)  , P  A   ,P(B)  , then A and B
6 3 4
are
(A) mutually exclusive (B) dependent
(C) independent (D) none of these
1  3p 1  p 1  2p
6. If , and are the probabilities of three mutually exclusive events,
3 2 2
then the set of all values of p is

1 1   1 2
(A) f (B) 2, 3 (C) [0, 1] (D)  3 , 3 
 
1
7. For independent events A1, . . ., An, P(Ai) = , i = 1, 2, . . ., n. Then the probability
i 1
that none of the events will occur is
(A) n/(n + 1) (B) (n – 1)/(n + 1) (C) 1/(n + 1) (D) n + ( 1/(n+1) )

8. If two events A and B are such that P  A  = 0.3, P(B) = 0.4 and  A  B = 0.5, then

 B 
P  =
 A  B 

1 1 3 2
(A) (B) (C) (D)
4 5 5 5

Page # 56
PROBABILITY
9. The probability that in a group of N (< 365)people, at least two will have the same
birthday is

 365 !  365N 365 !


(A) 1  (B) –1
 365  N ! 365 ! 365  N !
 365N 365 ! 1
 365  !
(C) 1 – (D)
 365  N !  365  N !  365 N
10. Let E and F be two independent events such that P(E) > P(F). The probability that
1 1
both E and F happen is and the probability that neither E nor F happens is ,
12 2
then
1 1 1 1
(A) P(E) = , P(F) = (B) P(E) = , P(F) =
3 4 2 6
1 1 1
(C) P(E) = 1, P(F) = (D) P E  , P F  
12 3 2
Independent Experiments
If two random experiments are performed separately i.e., their outcomes are
independent of each other, then the experiments are called independent experiments.
For example
(i) Consider the tossing of a coin twice, clearly the outcome for the second toss is
not effected by the result of the first toss. So the two tosses are independent random
experiments.
(ii) Consider the drawing of two balls one after the other, with replacement, from an
urn containing two or more balls. Then the two draws are independent of each other.
So considering each draw as an experiment, the two experiments are independent
random experiments.
(iii) In throwing of a die and a coin together or one after the other are independent
experiments.
Remarks
(i) In drawing of two cards, without replacement, from a well-shuffled ordinary pack
of 52-playing cards, the two draws are not independent experiments.
(ii) Let E1 and E2 be two independent random experiments.Let A be an event of
experiment
E1 only and B be an event of experiment E2 only. Then P(A occurs in E1 and B occurs
in E2) = P(A). P(B).

For example if a coin and a die are thrown together and A = {H}, B = {1, 2, 3, 4}, then
1 4 1
(head on coin and a number  4 on die) = P(A). P(B) =   .
2 6 3
Conditional probability
The probability of occurrence of an event B when it is known that some event A has
already occurred, is called the conditional probability and is denoted by P(B/A). The
symbol P(B/A) is usually read as probability of B, given A.Consider two events A and
B. When it is known that event ‘A’ has occurred, it means that sample space would
reduce to that sample points representing event A. Now for P(B/A) we must look for
the sample points representing the simultaneous occurrence of A and B i.e., sample
points in A  B .

Page # 57
PROBABILITY

n  A  B
n  A  B n(S) P(A  B)
 
 P(B/A) = n A n(A) P(A)
n(S)

P  A  B P  A  B
Thus P(B/A) = ,where 0 < P(A)  1. Similarly, P(A/B) = ,0 < P(B)  1
P A P B

P(A).P(B / A), P(A)  0


Hence, P  A  B  = 
P(B).P(A / B), P(B)  0
Ex.10 If a pair of dice is thrown and it is known that sum of the numbers is even, then find
the probability that the sum is less than 6.
Sol. Let A be the given event and let B be the event, whose probability is to be found. Then

 B P  B  A 4/36 2
Required probability P      .
A P(A) 18/36 9
Ex.11 Two dice are thrown. Then the probability that the numbers appeared has a
sum 8 if it is known that the second die always exhibits 4, is-
(A) 1/3 (B) 2/3 (C) 1/6 (D) 1/2
Sol. Let A = the event of occurrence of 4 always on the second die.
= {(1,4), (2,4), (3,4), (4,4), (5,4), (6,4)}
 n (A) = 6
and B = the event of occurrence of such numbers on both dice whose sum is
8 = {(4,4)}
Thus A  B = A  {(4,4)} = {(4,4)}
 n (A  B) = 1
n( A  B) 1
 P (B/A) = n( A)
=
6

DRILL - IV

1. Let P (A) = 0.4 and P (B/A) = 0.5. The probability P ( A  B ) is equal to-
(A) 0.8 (B) 0.7 (C) 0.6 (D) None of these
2. A pair of dice is thrown. If 5 appears on at least one of the dice, then the
probability that the sum is 10 or greater, is-
(A) 11/36 (B) 2/9 (C) 3/11 (D) 1/12
3. A pair of dice is thrown. If the two numbers appearing on them are different,
the probability that the sum is 6, is-
(A) 2/15 (B) 1/9 (C) 5/36 (D) 1/12
4. Two dice are thrown together. If 3 appears on at least one of the dice, then what
is the probability that the sum is greater than 9-
(A) 1/4 (B) 3/11 (C) 5/11 (D) zero
5. In a certain town, 40% of the people have brown hair, 25% have brown eyes and
15% have both brown hair and brown eyes. If a person selected at random has brown
hair, the probability that he also has brown eyes is-
(A) 2/5 (B) 1/4 (C) 1/2 (D) 3/8

Page # 58
PROBABILITY
6. A bag contains 7 red and 3 black balls. Three balls are drawn at random from
the bag one after the other. The probability that the first two are red and the
third is black is-
(A) 21/40 (B) 1/5 (C) 7/50 (D) 7/40
7. If A and B are two events such that P(A) > 0, and P( B)  1 , then P(A / B ) is equal to

P(A) 1  P(A  B)
(A) 1 – P(A/B) (B) 1 – P(A /B) (C) (D) P(B)
P(B)

2 1 1
8. If P(A  B) = , P(A B) = and P(A) = then-
3 6 3
(A) A and B are independent events (B) A and B are disjoint events
(C) A and B are dependent events (D) none of the above

Independent Events
Pairwise Independent Events
Two events A and B are said to be independent if occurrence or non–occurrence of
one does not affect the occurrence or non–occurrence of the other,
i.e., P(B/A) = P(B), P(A)  0.

P  A  B
 P(B/A) =  P(B)  P(A  B) = P(A). P(B)
P(A)

If the events are not independent, then they are said to be dependent.
Ex.12 Three coins are tossed together. What is the probability that first shows head,
second shows tail & third shows head ?
(A) 1/8 (B) 1/4 (C) 1/2 (D) 3/4
Sol. Let A,B,C denote three given component events which are mutually independent,
so P (ABC) = P (A). P (B). P (C)

1 1 1 1
= . . = Ans.[A]
2 2 2 8
Ex.13 A bag contains 4 red and 4 blue balls . Four balls are drawn one by one from
the bag then the probability that the drawn balls are in alternate colour is-
(A) 2/3 (B) 3/35 (C) 6/35 (D) 3/4
Sol. E1 = Event that first drawn ball is red, second is blue and so on.
E2 = Event that first drawn ball is blue second is red and so on.

4 4 3 3 4 4 3 3
 P (E1) = . . . & P(E2) = . . .
8 7 6 5 8 7 6 5
 P (E) = P (E1) + P (E2)
4 4 3 3 4 4 3 3 6
= . . . + . . . =
8 7 6 5 8 7 6 5 35
Ex.14 If A and B are independent events. Then show that the following pairs of events are
also independent (i) A and B (ii) A  and B

Sol. (i) P  A  B   P  B   P  A  B   P  B   P(A)P(B)

Page # 59
PROBABILITY

( A and B are independent) = P(B) (1 – P(A)) = P(B) P  A  .

 
(ii) P  A   B   P  A  B   1  P  A  B  = 1  P(A)  P(B)  P(A).P(B)

= 1 – P(A) – P(B) + P(A). P(B) = (1 – P(A)) (1 – P(B)) = P  A  . P  B  .

Mutually independent Events


Three events A, B and C are said to be mutually independent if,

P  A  B = P(A). P(B), P  A  C  = P(A). P(C), P  B  C  = P(B). P(C)

and P  A  B  C  = P(A). P(B). P(C)

These events would be said to be pair–wise independent if,

P  A  B = P(A). P(B), P  B  C  = P(B). P(C) and P  A  C  = P(A). P(C).


Thus mutually independent events are pair–wise independent but the converse may
not be true.
Ex.15 A lot contains 50 defective and 50 non-defective bulbs. Two bulbs are drawn at random,
one at a time with replacement. The events A, B and C are defined as under:
A = {The first bulb is defective}, B = {The second bulb is non-defective}
C = {The two bulbs are either both defective or both non-defective}
Catogorize the events A, B and C to be pairwise independent or mutually independent.
1 1 1 1 1 1 1 1 1
Sol. P(A) =  1  , P(B) = 1  and P(C) =    
2 2 2 2 2 2 2 2 2
P  A  B  P (the first bulb in defective and the second bulb is non-defective)
1 1 1
=   .
2 2 4
1 1 1
P  B  C  = P(both the bulbs are non-defective) =  
2 2 4
1 1 1
P  C  A  = P (both the bulbs are defective) =  
2 2 4

P  A  B  C   P    0

As P  A  B   P(A).P(B), P  B  C   P  B  .P  C  and P  C  A   P(C).P(A) , the events


A, B and C are pairwise independent.
Since P  A  B  C   0  P(A).P(B).P(C) , A, B and C are not mutually independent.
Ex.16 A problem of mathematics is given to three students A, B and C. Whose chances
of solving it are 1/2, 1/3, 1/4 respectively. Then probability that the problem is
solved is-
(A) 1/4 (B) 1/2 (C) 3/4 (D) 3/16
Sol. Obviously the events of solving the problem by A,B and C are independent.
Therefore required probability
 1  1  1  1 2 3 3
= 1 –  1   1   1    = 1 – . . =
 2  3  4  2 3 4 4

Page # 60
PROBABILITY

Rule of Multiplication

Suppose A1, A2, . . . , An be n events such that P  A1  A 2  . . .  A n   0 , then

 n 
 
P  A i   P  A1  .P  A2 / A1  .P  A 3 / A1  A 2  . . .P  A n / A1  A 2  . . .  A n 1 

 i 1 
Ex.17 P1, P2, . . . , P8 an eight players participating in a tournament. If i < j, then Pi will win
the match against Pj. Players are paired up randomly for first round and winners of
this round again paired up for the second round and so on. Find the probability that
P4 reaches in the final.
Sol. Let A1 be the event that in the first round the four winners are P1, P4, Pi, Pj, where i
 {2, 3}, j  {5, 6, 7}and let A2 be the event that out of the four winners in the first
round, P1 and P4 reaches in the final.
The event A1 will occur, if P4 plays with any of P5, P6, P7 or P8 (say with P6) and P1, P2
and P3 are not paired with P5, P7 and P8. Further A2 will occur if P1 plays with Pi.

 A2 
The required probability = P  A1  A 2  = P(A1). P  A  .
 1

 6 
4   3
  23 3  1
  4
= 8 4 = .
35
4 2 4 2 2 2

(Here we have used the concept of division into groups).

DRILL - V
1. If A and B are two independent events then P (A  B) equals-
(A) P(A) + P (B) (B) P (A) . P (B)
(C) P (A/B) (D) None of these
2. If A and B are two independent events, then the probability that only one of A and
B occur is-
(A) P (A) + P (B) – 2P (A  B) (B) P (A) + P(B) – P (A  B)
(C) P (A) + P (B) (D) None of these
3. For two given events A and B, the relation P (AB) = P (A) P(B) implies that A and
B are-
(A) Independent (B) Mutually exclusive
(C) Dependent (D) None of these
4. If p 1 and p 2 are the probabilities of two independent events then
(1 – p 1 – p 2 + p 1p 2) is the probability of -
(A) Their joint occurrence (B) Occurrence of at least one
(C) Occurrence of None of these (D) Occurrence of only one
5. A coin is tossed three times. The probability of getting all heads or tails only
is-
(A) 0 (B) 1/2 (C) 1/4 (D) 1

Page # 61
PROBABILITY
6. A coin is tossed four times then the probability of obtaining at least one tail is-
(A) 1/16 (B) 14/16 (C) 15/16 (D) 1/4
7. The probability of getting head and tail alternatively in three throws of a coin
(or in a throw of three coins) is-
(A) 1/3 (B) 1/4 (C) 1/5 (D) 3/5
8. The probability of not getting tail in the first two times and getting a tail in the
third time by tossing a coin continuously is-
(A) 1/4 (B) 1/8 (C) 3/8 (D) 7/8
9. From a pack of 52 cards two cards are drawn in succession the first having been
replaced before the second is drawn. The probability that the first is a diamond
and the second is a king, is-
(A) 1/52 (B) 1/13 (C) 1/4 (D) 4/13
10. If two cards are drawn from a pack of card one by one. If first drawn card is
replaced then the probability of getting two jacks is-
(A) 1/221 (B) 1/169 (C) 12/221 (D) 4/663
11. A bag contains 6 black and 5 white balls, while the second bag contains 7 black
and 4 white balls. Two balls are drawn one from each bag, the probability of both
being black is-
(A) 2/11 (B) 11/13 (C) 20/121 (D)42/121
12. The probability that A will pass in a examination is 2/5 and the probability that
B will fail in the same examination is 3/4. The probability that only one of them
will pass in the examination is-
(A) 3/20 (B) 3/10 (C) 9/20 (D) None of these
13. For solving a problem, odds against to A are 4 : 3 and odds in favour to B are
7: 5. The probability that the problem will not be solved is-
(A) 16/21 (B) 5/21 (C) 43/84 (D) 45/84
14. The probability of solving a problem by A and B are 1/4 and 2/3 respectively. If
A and B work independently, then the probability that the problem will be solved
by both of them is-
(A) 1/6 (B) 3/4 (C) 1/3 (D)11/12
15. The probabilities that three boys will pass an examination are 1/6, 1/4 and 1/3
respectively. The probability that exactly one boy will pass the examination is -
(A) 31/72 (B) 7/12 (C) 41/72 (D) 11/12
16. If A and B are any two events such that P (A + B) = 5/6, P (AB) = 1/3, P( B ) = 1/
2, then the events A and B are-
(A) independent (B) dependent
(C) mutually exclusive (D) exhaustive
17. A card is drawn from a pack of playing cards. It is replaced in the pack and the
pack is shuffled, and again a card is drawn. This process as repeated six times,
then probability of getting in sequence2 heart, 2 diamond and 2 black cards is
(A) (1/4)4 (B) (1/4)5
(C) (1/4)6 (D) None of these
18. A man and a woman appear in an interview for two vacancies in the same post.
The probability of man’s selection is 1/4 and that of the woman’s selection is 1/
3. What is the probability that none of them will be selected-
(A) 1/2 (B) 1/12 (C) 1/4 (D) None of these

Page # 62
PROBABILITY
19. If the probabilities of three persons A, B & C hitting a target are 3/5, 2/5 and
3/4 respectively. If they hit at a time then the probability that two persons hit
the target is-
(A) 9/50 (B) 9/20 (C) 11/20 (D) 41/50
20. A bag contains 4 black and 3 white balls. Two- two balls are drawn two times.
If balls are not replace once it is drawn then the probability that first two balls
are black and second two balls are white is-
(A) 4/49 (B) 2/35 (C) 1/35 (D) 3/35
21. The probability that Krishna will be alive 10 years hence is 7/15 and Hari will
be alive is 7/10. The probability that both Krishna and Hari will be dead 10 years
hence is-
(A) 21/150 (B) 24/150 (C) 49/150 (D) 56/150
22. From the records of a hospital, it is found that 20% patients died with the
disease cancer. If two patients with cancer are admitted to hospital; then
probability that at least one patient will be cured, is-
(A) 16/25 (B) 24/25 (C) 9/25 (D) None of these
23. A draws two cards one by one (replacing previous one) from a pack of cards and
B throws two dice together. The probability that both cards of A are of the same
suit and the sum of digits of B is 6, will be-
(A) 1/4 (B) 1/44 (C) 5/144 (D) 7/144
24. India plays two matches each with West Indies and Australia. In any match
the probability to get 0,1 and 2 point by India are 0.45, 0.05 and 0.50 respectively.
If the results are independent, then the probability that India gets at least 7
points is-
(A) 0.8750 (B) 0.0875 (C) 0.6250 (D) 0.0250
25. The probability that a man will remain alive for the next 25 years is 4/5 and
the probability that his wife will remain alive for the same 25 years is 3/4. The
probability that at least one of them will be alive 25 years hence, is-
(A) 19/20 (B) 3/5 (C) 3/20 (D) None of these
26. A piece of equipment will function only when all the three components A, B and
C are working. the probability of A failing during one year is 0.15, that of B is
0.05 and that of C is 0.10. The probability that the equipment will fail before the
end of the year is-
(A) 0.72675 (B) 0.27325 (C) 1 (D) 0.95
27. A box contains 15 tickets numbered 1, 2,...., 15. Seven tickets are drawn at
random one after the other with replacement. The probability that every time the
greatest number on a drawn ticket is 9, is-
(A) (9/10)6 (B) (8/15)7 (C) (3/5)7 (D) None of these
28. If A and B are two events, then which is not correct-
c
(A) P(AB ) = P(A) – P(AB)
c c
(B) P(AB ) + P(A B) = P(A  B) – P (AB)
(C) P(AB) = P(A) + P(B) – P (A  B)
(D) If A and B are independent events then P(AB) = 0
Total Probability Theorem
Suppose A1, A2 . . . , An are mutually exclusive and exhaustive events of a random
experiment and ‘B’ be an events of the same experiment which can occur together
with A1 or A2 or ..... or An, then

Page # 63
PROBABILITY

n
B    B  Ai 
i 1

n
 P  B    P  B  Ai  , as events B  Ai , i = 1, 2, . . ., n are exclusive.
i 1

n
 P(B) =  P  B / Ai .P  Ai  .
i 1

Ex.18 Two sets of candidates are competing for the positions on the board of directors of a
company. The probabilities that the first and second sets will win are 0.6 and 0.4
respectively. If the first set wins, the probability of introducing a new product is 0.8,
and the corresponding probability, if the second set wins is 0.3. What is the probability
that the new product will be introduced?
Sol. Let A1 (A2) denotes the event that first (second) set wins and let B be the event that
a new product is introduced.
 P(A1) = 0.6, P(A2) = 0.4

 B   B 
P  A   0.8, P  A  = 0.3
 1  2

 B   B 
P(B) = P  B  A1   P  B  A 2   P  A1  .P    P  A2  P  .
 A1   A 2 
= 0.6  0.8  0.4  0.3  0.6 .
DRILL - VI
1. A purse contains 4 copper and 3 silver coins and another purse contains 6 copper and
2 silver coins. One coin is drawn from any one of these two purses. The probability
that it is a copper coin is
4 3 2 37
(A) (B) (C) (D)
7 4 7 56
2. A bag contains a large number of white and black marbles in equal proportions. Two
samples of 5 marbles are selected (with replacement) at random. The probability
that the first sample contains exactly 1 black marble, and the second sample contains
exactly 3 black marbles, is

25 15 15 35
(A) (B) (C) (D)
512 32 1024 256

BAYE’S THEOREM
Suppose A1, A2, . . ., An are mutually exclusive and exhaustive events of a random
experiment.
Then the conditional probability that Aj happens given that B has happened is given
by

P(Aj/B) =

P Aj B    
P A j .P B / A j  , where j = 1, 2, ....., n.
P B n
 P  A j  .P  B / A j 
j1

Page # 64
PROBABILITY
Ex.19 Box I contains 2 white and 3 red balls and box II contains 4 white and 5 red balls.
One ball is drawn at random from one of the boxes and is found to be red. Find the
probability that it was from box II.
Sol. Let A denote the event that the drawn ball is red
Let A1  The event that box I is selected and let A2  The event that box II is selected

 A  1 5
P  A2  .P   .
A2  2 9 25
 A2   
 P  = 1 5 1 3 32
 A   A   A  .  .
P  A1  P    P  A 2  .P   2 9 2 5
 A1   A2 

DRILL - VII
1. A bag A contains 2 white and 3 red balls and bag B contains 4 white and 5 red
balls. One ball is drawn at random from a randomly chosen bag and is found to
be red. The probability that it was drawn from bag B was-
(A) 5/14 (B) 5/16 (C) 5/18 (D) 25/52
2. A man is known to speak the truth 3 out of 4 times. He throws a die and reports
that it is a six. The probability that it is actually a six, is-
(A) 3/8 (B) 1/5 (C) 3/4 (D) None of these
1, 1, 1
3. The probability of defective screws in three boxes A,B,C are respectively. A
5 6 7
box is selected at random and a screw drawn from it at random is found to be
defective. Then the probability that it came from box A is-
(A) 16/29 (B) 1/15 (C) 27/59 (D) 42/107
4. A company has two plants to manufacture televisions. Plant I manufacture 70% of
televisions and plant II manufacture 30%. At plant I, 80% of the televisions are rated
as of standard quality and at plant II, 90% of the televisions are rated as of standard
quality. A television is chosen at random and is found to be of standard quality. The
probability that it has come from plant II is
17 27 3 9
(A) (B) (C) (D)
50 83 5 83
5. Fifteen coupons are numbered 1,2,........15, respectively. Seven coupons are selected
at random one at a time with replacement. The probability that the largest number
appearing on a selected coupon is 9, is
6 7 7 7 7
 9   8  3  9   8 
(A)   (B)   (C)   (D)    
 16   15  5  15   15 
6. The chances of defective screws in three boxes A, B, C are 1/5, 1/6, 1/7, respectively.
A box is selected at random and a screw drawn from it at random is found to be
defective. Then the probability that it came from box A is
(A) 16/29 (B) 1/15 (C) 27/59 (D) 42/107

Binomial distribution for repeated trials


Let an experiment is repeated n times and probability of happening of any event
called success is p and not happening the event called failure is q = 1– p then
by binomial theorem.
(q + p)n = q n + nC1 qn–1 p +.....+ nCr qn–r pr
+....+ p n
Page # 65
PROBABILITY
Now probability of
(a) Occurrence of the event exactly r times
= nCr qn–rp r
(b) Occurrence of the event at least r times
= nCr qn–r p r + .....+ p n
(c) Occurrence of the event at the most r times
= qn + nC1 qn–1p+ ...+ nCr qn–r p r
Ex.20 Two dice are tossed four times find the probability of getting
(i) equal digits exactly two times
(ii) equal digits at least two times
(iii) equal digits at the most two times
Sol. Let A be the event of getting equal digits on the dice. Since number of exhaustive
6 1
cases is 36 and favourable cases is 6. Therefore P (A) = P = = ,
36 6
1 5
P( A ) = q = 1  =
6 6
Hence by Binomial theorem, we have
(5/6 + 1/6) 4 = (5/6)4 + 4C1 (5/6)3 (1/6) + 4C2 (5/6)2 (1/6)2 + 4C3 (5/6) (1/6) 3
+ (1/6)4
Thus from above result, we have
(i) Probability of getting equal digits exactly two times

25
= 4C2 (5/6)2 (1/6)2 =
216
(ii) Probability of getting equal digits at least two times.
= 4C2 (5/6)2 (1/6)2 + 4C3 (5/6) (1/6) 3 + (1/6)4

25 20 1 171
= + + =
216 1296 1296 1296
(iii) Probability of getting equal digits at the most two times
= (5/6)4 + 4C1 (5/6)3 (1/6) + 4C2 (5/6)2 (1/6)2

625 500 150 1275


= + + =
1296 1296 1296 1296

DRILL - VIII
1. India and Pakistan play a 5 match test series of hockey, the probability that India
wins at least three matches is-

1 3 4 1
(A) (B) (C) (D)
2 5 5 3
2. One hundred identical coins, each with probability, p, of showing up heads are tossed
once. If 0 < p < 1 and the probability of heads showing on 50 coins is equal to that of
heads showing on 51 coins, then the value of p is
1 49 50 51
(A) (B) (C) (D)
2 101 101 101

Page # 66
PROBABILITY
3
4. The probability that a man can hit a target is . He tries 5 times. The probability
4
that he will hit the target at least three times is
291 371 471 459
(A) (B) (C) (D)
364 461 502 512
5. A die is thrown 7 times. The chance that an odd number turns up at least 4 times, is

1 1 1 3
(A) (B) (C) (D)
4 2 8 4
6. India plays two matches each with West Indies and Australia. In any match the
probabilities of India getting points 0, 1 and 2 are 0. 45, 0.05 and 0.50 respectively.
Assuming that the outcomes are independent, the probability of India getting at least
7 points is
(A) 0.8750 (B) 0.0875 (C) 0.0625 (D) 0.0250
7. The probability of having at least one tail in 4 throws with a coin is-
15 1 1
(A) (B) (C) (D) 1
16 16 4
8. The probability that an event A happens in one trial of an experiment is 0.4.
Three independent trails of the experiment are performed. The probability that
the event A happens at least once is-
(A) 0.936 (B) 0.784 (C) 0.904 (D) None of these
9. A pair of dice is thrown four times. If getting the same number on both dice
is considered as a success, the probability of getting two success is-
(A) 20/216 (B) 25/216 (C) 19/216 (D) None of these
10. A cube is thrown 6 times, then probability of getting the digits 2 and 4 exactly
three times each is-
(A) 1/5184 (B) 5/11664 (C) 1/46656 (D) 3/11664
11. A box of 100 bulbs has 90 bulbs right then in a sample of 8 bulbs, the probability
that at least one bulb is defective is-
8 8 8 8
 9   9   1   1 
(A) 1 –   (B)  10  (C)   (D) 1–  
 10     10   10 
12. The odds in favour of escape of an enemy ship are 4 : 1 . The probability that at
least one ship out of three ships gets destroyed is-
(A) 1/125 (B) 16/125 (C) 61/125 (D) 64/125
13. If X is binomial variate with parameters n and p, where 0 < p <1 such that
P(X  r)
P (X  n  r)
is independent of n a nd r, then p equ als-

(A) 1/2 (B) 1/3 (C) 1/4 (D) none of these


14. Let X denote the number of times heads occur in n tosses of a fair coin. If
P(X = 4), P (X = 5) and P (X = 6) are in AP; the value of n is-
(A) 7 (B) 10 (C) 12 (D) 8
15. If X follows a binomial distribution with parameters n = 8 and p = 1/2, then P
(| X – 4 | < 2) equals-

118 119 117


(A) (B) (C) (D) none
128 128 128

Page # 67
PROBABILITY
16. A random variable has the following probability distribution-
X : 0 1 2 3 4 5 6 7
P(X) : 0 2p 2p 3p p2 2p2 7p2 2p
The value of p is
(A) 1/10 (B) –1 (C) –1/10 (D) none of these
17. A random variable X has the distribution-
X 2 3 4
P(X = x) 0.3 0.4 0.3
Then, variance of the distribution is
(A) 0.6 (B) 0.7 (C) 0.77 (D) 1.55
18. A fair die is thrown twenty times. The probability that on the tenth throw the
fourth six appears is-
20
C10  56 120  57
(A) (B)
620 610

84  56
(C) (D) none of these
610
(e) Probability regarding n letters and their envelopes:
If n letters corresponding to n envelopes are placed in the envelopes at
random, then
1
(i) Probability that all letters are in right envelopes =
n!
1
(ii) Probability that all letters are not in right envelopes = 1 –
n!
(iii) Probability that no letter is in right envelope
1 1 1 1
= – + – .....+ (–1) n
2! 3! 4! n!
(iv) Probability that exactly r letters are in right envelopes

n !  1  1  1  ........  (1)n  r 1 
=  
r !  2! 3! 4! (n  r)! 
Ex.21 There are four letters and four envelopes, the letters are placed into the
envelopes at random, the probability that all letters are placed in the wrong
envelopes-
(A) 1/24 (B) 23/24 (C) 3/8 (D) 5/8
Sol. We know from the above given formula that probability that no letter is in right
envelop out of n letters and n envelopes is given by

1 1 1 n 1 
=  2!  3!  4!  ........  (1) n ! 
 
Since all 4 letters are to be placed in wrong envelopes then required probability
1 1 1 1 1 1 3
=     =   =
 2! 3! 4!  2 6 24 8

Page # 68
PROBABILITY

DRILL - IX
1. Three letters are written to three different persons and their addresses are
written of three envelopes. The probability that letters are placed in right
envelopes without seeing the addresses is -
(A) 1/27 (B) 1/6 (C) 1/9 (D) None of these
2. 3 letters are placed in 3 envelopes randomly. The probability that all letters are
not in right envelopes is-
(A) 1/6 (B) 1/2 (C) 1/3 (D) 5/6
3. There are n letters and n addressed envelopes. The probability that all the
letters are not kept in the right envelope, is-
1 1 1
(A) (B) 1– (C) 1 – (D) None of these
n! n! n

SOLVED EXAMPLES
Ex.1 If m different cards are placed at random and independently into n boxes lying in a
straight line (n > m), find the probability that the cards go into m adjacent boxes.
Sol. m cards can be placed into n boxes independently in nm ways.
 n(S) = nm
Now m adjacent boxes can be chosen in n – m + 1 ways. In each of these the cards
can be placed into m! ways.
total number of ways in which the cards can be placed in m consecutive boxes
= (n – m + 1). m!
m!  n  m  1
Required probability =
nm
Ex.2 Out of 21 tickets consecutively numbered, three are drawn at random. Find the
probability that the numbers on them are in A.P.
21
Sol. Any three tickets out of 21 tickets can be chosen is C3 ways. For the favourable
a c
choice if the choose numbers are a, b and c, a < b < c, then we should have  b.
2
Obviously either both a and c are even or both are odd and then b is fixed. Hence for
the favourable choice we have to choose two numbers from 1 to 21, which are either
both even or both odd. This can be done in 11C2 + 10C2 ways.

11
C2 10 C2 10
Hence required probability = 
21
C3 133 .

Ex.3 A has 3 shares in a lottery containing 3 prizes and 9 blanks. B has 2 shares in a
lottery containing 2 prizes and 6 blanks. Compare their chances of success.
Sol. Let E1 be the event of success of A and let E2 be the event of success of B
Since A has 3 shares in a lottery containing 3 prizes and 9-blanks, A will draw 3
tickets out of 12 tickets (containing 3 prizes and 9 blanks). A will get success if he
draws atleast one prize out of 3 draws.
9
C 21
 P  E1   12 3 
C3 55
21 34
 P  E1   1  
55 55
Page # 69
PROBABILITY
6
Again, P  E2 C2 15
  
8 28
C2
15 13
 P(E2) = 1 – 28  28

P  E1  34 28 952
   
P  E2  55 13 715
 P(E1) : P(E2) = 952 : 715.

Ex.4 A coin is tossed m + n times (m > n). Show that the probability of at least m consecutive
n2
heads come up is .
2m 1
Sol. Let H, T and S be the events “head turns up”, “tail turns up” and “head or tail turns
up”
1
Then P(H) = P(T) = and P(S) = 1
2
Since the given event is “at least m consecutive heads turn up”, therefore in any
favorable out come there are m consecutive heads and the rest are any of head or
tail
Consider the events
 
  1 1
H,H,H,. . .,H
A1 =    ,S,S,S,.

. .,S
 with P(A1) = .1n 
m

 m times n times 
 2 2m

 
  1 1 n 1 1
T,H,H,H,. . .,H ,S,S,S,. . .,S
A2 =       with P(A2) = . m .1 

 m times n 1times  
2 2 2 1
m

 
  1 1 n 2 1
S, T,H,H,H,. . .,H ,S,S,S,. . .,S
A3 =       with P(A3) = 1. . m .1 

 m times n  2 times  
2 2 2 1
m

 
  1 1 1
S,S,S,. . .,S , T,H,H,H,. . .,H
. . . An + 1 =      With P(An ) = 1 n – 1. .
m

 n 1 times
 m times 

+1 2 2 2 1
m

The given event is A1  A 2  A 3  A n 1 . As A 1, A 2, A 3, . . ., A n + 1


are pair – wise
mutually exclusive.
The required probability
1 1 1 1
  . . .
m m 1 m 1 m 1
= P(A1) + P(A2) + P(A3) + . . . + P(An + 1) = 2 2
2  2 
n  times
1 n 2n
=   .
m m 1
2 2 2m 1

Page # 70
PROBABILITY
Ex.5 There are four six faced dice such that each of two dice bears the numbers 0, 1, 2, 3,
4 and 5 and the other two dice are ordinary dice bearing numbers 1, 2, 3, 4, 5 and 6.
If all the four dice are thrown, find the probability that the total of numbers coming
up on all the dice is 10.
Sol. Total number of sample points in the sample space = 64 = 1296
Number of sample points in favour of the event
= Coefficient of x10 in the expansion of (1 + x + x2 + . . . + x5)2 (x + x2 + . . . + x6)2
= Coefficient of x10 in the expansion of x2(1 + x + x2 + . . . + x5)4
= Coefficient of x8 in the expansion of (1 + x + x2 + . . . + x5)4

4
 1  x6 
= Coefficient of x in the expansionof 
8 

 1 x 

= Coefficient of x8 in the expansion of (1 – x6)4 (1 – x)–4

 45 2 45 6 3 
= Coefficient of x8 in the expansion of (1 – 4x6) 1  4x  x  x  ...
 2! 3! 

125
= 1 11 C8  4 5 C2  125 .  Required probability = .
1296
Ex.6 A die is thrown 7 times. What is the probability that an odd number turns up (i)
exactly 4 times (ii) atleast 4 times.
3 1 1 1
Sol. Probability of success =  p = , q=
6 2 2 2
4 3
1 1 35
(i) For exactly four successes, required probability = C4.   .   
7
 2  2 128

(ii) For atleast four successes, required probability

4 3 5 2 6 1 7
1 1 7 1 1 7 1 1 7 1
= C47
 2  .  2   C5  2  .  2   C6  2  .  2   C7  2 
             

35 21 7 1 64 1
=      .
128 128 128 128 128 2
Ex.7 If m things are distributed among ‘a’ men and ‘b’ women, show that the probability
m m
1   b  a    b  a  
that the number of things received by men is odd, is 2  .
  b  a m 

a
Sol. A particular thing is received by a man with probability p = and by a woman with
a b
b
probability q = . If distributing a single object is an experiment, then this
a b
experiment is repeated m time.

The required probability = m C1 . p. qm – 1 + m


C3 . p3. qm – 3 + mC5 . p5. qm – 5 + . . .

Page # 71
PROBABILITY

m m m
 q  p m   q  p m 1  ba  1   b  a    b  a  
= = 2 1   b  a  =  .
2     2   b  a m 

8
Ex.8 An artillery target may be either at point A with probability or at point B with
9
1
probability . We have 21 shells each of which can be fixed either at point A or B.
9
1
Each shell may hit the target independently of the other shell with probability .
2
How many shells must be fired at point A to hit the target with maximum probability?
Sol. Let E denote the event that the target is hit when x shells are fired at point A. Let E1
(E2) denote the event that the artillery target is at point A  B  .

8 1
We have P(E1) = , P(E2) = .
9 9
x 21 x
E 1  E  1
 P  E   1   2  and P  E   1   2 
 1    2  
 x  21 x 
8 1   1    1 1   1  
Now P(E) =    
9   2   9   2  
x 21 x
d 81 1 1 
 dx  P(E)      n 2       n2 
9 2 9  2 
 
d
Now we must have
dx
 P  E  0
d2
 x = 12, also  P  E  0 at x = 12
dx 2
Ex.9 Let p be the probability that a man aged x years will die within a year. Let A1, A2, . . , An
be n men each aged x years. Find the probability that out of these n men A1 will die
with in a year and is first to die.
Sol. P(no one among A1, A2 . . . , An dies within a year) = (1 – p)n
P (at least one among A1, A2, . . ., An dies within a year) = 1 – (1 – p)n

1 n
P(A1dies within a year and is first to die) = 1  1  p   .
n  
Ex.10 Each of three bags A, B, C contains white balls and black balls. A has a1 white & b1
black, B has a2 white & b2 black and C has a3 white & b3 black balls. A ball is drawn
from a bag and found to be white. What are the probabilities that the ball is from bag
A, B and C.
Sol. Let A1, A2 and A3 be the events that the bag picked is A, B and C respectively.
Let E be the event that a white ball is drawn.
We are supposed to find P(A1/E), P(A2/E), P(A3/E).

 A1 
P =
 E 

Page # 72
PROBABILITY

P  A1  E   Probability that bag A is chosen and white ball is drawn 


 
P(E)  Probability that a bag is chosen at random and a white ball is drawn

P  A1  .P  E / A1 
= P A .P E / A  P A .P E / A  P A .P E / A
 1  1  2  2  3  3

1 a1
.
3 a1  b1 p1
 ak
= 1  a1 a2 a3  p1  p2  p3 , Pk =
.    a k  bk , k = 1, 2, 3.
3  a1  b1 a 2  b2 a 3  b3 

p2 p3
Similarly, P(A2/E) = p  p  p , P(A3/E) = p  p  p .
1 2 3 1 2 3

Ex.11 The probability that at least one of A and B occurs is 0.6. If A and B occur simultaneously
with probability 0.3, then find the value of P( A)  P ( B) .

Sol. We have P( A  B)  0.6 and P( A  B)  0.3

P(A) + P(B) = P( A  B) + P( A  B) = 0.6 + 0.3 = 0.9

 P( A)  P(B)  1  P( A)  1  P( B)  2  0.9  1.1


Ex.12 There are n students in a class and probability that exactly  out of n pass the

examination is directly proportional to 2 0    n  .


(i) Find out the probability that a student selected at random was passed the
examination
(ii) If a selected student has been found to pass the examination then find out the
probability that he is the only student to have passed the examination.
Sol. Let E  be the event that exactly  out of n pass the examinations and let A be the
event that a student selected randomly pass the examination.

 P E   2

2
 P  E    k (k is proportionality constant)
Since E0, E1, E2, . . . , En are mutually exclusive and exhaustive events.
 P(E0) + P(E1) + P(E2) + . . . + P(En) = 1
 0 + k (1)2 + k (2)2 + . . . + k (n2) = 1

6
 k = n  n  1 2n  1 . . . (i)

n
n
 k n 3 3  n  1
(i) P(A)   P  E  P  A / E  =  k2     
1
n n
1
2  2n  1
 0

Page # 73
PROBABILITY

A
P  E1  .P   2  A  P  A  E  
E
(ii) P  1    E1    2 
 P  E   P  E   n .
 
A P  A  n  n  1 

Ex.13 Let A and B be two independent witnesses in a case. The probability that A will speak
the truth is x and the probability that B will speak the truth is y. A and B agree in a
certain statement. Show that the probability that the statement is true is
xy
1  x  y  2xy .
Sol. Let E1 be the event that both A and B speak the truth, E2 be the event that both A and
B tell a lie and E be the event that A and B agree in a certain statement. Let C be the
event that A speaks the truth and D be the event that B speaks the truth.

 E1 = C  D and E2 = C  D . P(E1) = P  C  D  = P(C) P(D) = xy and

P(E2) = P  C  D  = P  C  P  D  = (1 – x) (1 – y) = 1 – x – y + xy

E 
Now P   = probability that A and B will agree when both of them speak the truth = 1
 E1 

 E 
and P  E  = probability that A and B will agree when both of them tell a lie = 1.
 2

 E1 
Clearly,  E  be the event that the statement is true.
 

 E1  P  E1  .P  E / E1 
 P  E   P  E  .P  E / E   P  E  P  E / E 
  1 1 2 2

xy.1 xy
=  .
xy.1  1  x  y  xy  .1 1  x  y  2xy

Ex.14 Find the minimum number of tosses of a pair of dice, so that the probability of getting
the sum of the numbers on the dice equal to 7 on atleast one toss, is greater than
0.95. (Given log102 = 0.3010, log103= 0.4771).
Sol. n(S) = 36
Let E be the event of getting the sum of digits on the dice equal to 7, then n(E) = 6.

6 1 5
P(E) =   p , then P  E  = q =
36 6 6
probability of not throwing the sum 7 in first m trails = qm .

m
 5 m
 P(at least one 7 in m throws) = 1 – q = 1 –  6  .
 

m m
5  5
According to the question 1     0.95   6   0.05
6  

Page # 74
PROBABILITY

 m log10 5  log10 6  log10 1  log10 20


 m > 16.44
Hence, the least number of trails = 17.
Ex.15 Two teams A and B play a tournment. The first one to win (n + 1) games, win the
series. The probability that A wins a game is p and that B wins a game is q (no ties).
Find the probability that A wins the series. Hence or otherwise prove that
n
1
 n  r Cr . 2 n  r  1 .
r 0

Sol. A wins the series, if out of first n + r games A wins n games, 0  r  n and wins the
(n + r + 1)th game.

 n 
 P(A) =   n  r Cn .q r .pn  . p (where p + q = 1)
 r 0 
n
Similarly, P(B) =   n r Cn  .q n1.pr
r 0
Now P(A) + P(B) = 1
n
  q r .pn 1  q n1.pr  nr Cn  1 . . . (i)
r 0
n
1 1
Now put p = q =
2 from (i),
  n  r Cn  . 2 n  r  1 .
r 0
Ex.16 Suppose the probability for A to win a game against B is 0.4. If A has an option of
playing either a" best of 3 games " or a " best of 5 games " match against B, which
option should he choose so that the probability of his winning the match is higher ?
(No game ends in a draw).
Sol. The probability p1 of winning the best three games is
= the prob. of winning two games + the prob. of winning three games.
= 3C2 (0.6) (0.4)2 + 3C3 (0.4)3
[Using Binomial distribution]
Similarly the probability of winning the best five games is p2 = the prob of winning
three games + the prob. of winning four games + the prob. of winning 5 games.
= 5c3 (0.6)2 (0.4)3 + 5c4 (0.6) (0.4)4 + 5c5 (0.4)5
We have p1 = 0.288 + 0.064 = 0.352
and p2 = 0.2304 + 0.0768 + 0.01024 = 0.31744
As p1 > p2
A must choose the first offer i.e., best of three games.
Ex.17 Suppose that it is 9 to 7 against a person A who is now 35 years of age living till he
is 65 and 3 to 2 against a person B now 45 living till he is 75, then find the chance
that one atleast of these persons will be alive 30 years hence.

7 2
Sol. A/q, P(A) = , P(B) = ;
16 5
Required probability = P(A) P(B) + P(A) P(B) + P( A ) P(B)

Page # 75
PROBABILITY

7 3 7 2 9 2 21 14  18 53
=      = 
16 5 16 5 16 5 80 80
Ex.18 If a pair of fair dice is rolled 5 times, then find out the probability that 3 times we get
sum more than 9.
Sol. A = event of getting sum is more than 9 exactly six times
{4, 6), (6, 4), (5, 6), (6, 5), (5, 5), (6, 6)}.
6 1 5
p(A) =  ; P (A) =
36 6 6
3 2
1 5
5
250
Required probability = C3     =
6 6 65
Ex.19 In a certain town, 40% of the people have brown hair, 25% have brown eyes and 15%
have both brown hair and brown eyes. If a person selected at random from the town,
having brown hair, then find the probability that he also has brown eyes.
Sol. Let H & E be the brown hair and brown eyes of a person.
A/q, P(H) = 4/10, P(E) = 25/100 = 1/4, and P(H  E) = 3/20

3
P(E  H) 20 3
Required probability = P(E/H) =  
P(H) 4 8
10
Ex.20 Four cards are chosen at random one by one, without replacement from a well
shuffled pack of 52 playing cards. Show that the probability that all the chosen cards
1
are aces, is 270725 .
 
4
C4 1
Sol. P= 52

C4 2 13 17  49
Ex.21 Cards are drawn one by one (without replacement) at random from a well shuffled
full pack of 52 playing cards until two aces are obtained for the first time. If N is the
number of cards required to be drawn, then find P(N = n).
Sol. P(N = n) = P (Event that one ace is obtained) among N- 1 cards
4
C1 48 Cn 2 3
P(Event that nth card is an ace) = 52

Cn 1 53  n

(n  1)(52  n)(51  n)
= 50  49 17 13
Ex.22 A bag contains 10 fair coins and 25 coins having heads on both sides. A coin is
selected at random and tossed. If it gives head, then find out the probability that it
was a fair coin.
Sol. E : The tossed coin gives head ;
E1 : The coin selected is fair.
E2 : The coin selected is not fair.

10 1

P(E1 ) P(E /E1 ) 35 2 1
 
P(F/E) = P(E1 ) P(E /E1 )  P(E2 ) P(E /E2 ) 10 1 25 .
  1 6
35 2 35

Page # 76
PROBABILITY

EXERCISES
LEVEL - I
SINGLE CORRECT TYPE QUESTIONS
1. Three natural numbers are taken at random from the set A ={ x| 1  x  100, x  N}.
The probability that the AM of the numbers taken is 75, is
77 25 74
C2 C2 C72
(A) 100 (B) 100 (C) 100 (D) None of these
C3 C3 C97
2. Let S be the universal set and n(X) = k. The probability of selecting two subsets A and
B of the set X such that B  A is

1 1 1 1
(A) (B) k (C) (D)
2 2 1 2k 3k
3. If ten objects are distributed at random among ten persons, the probability that at
least one of them will not get anything is

1010  10 1010  10! 1010  1 10!


(A) (B) (C) (D) .
1010 1010 1010 1010
4. 10 different books and 2 different pens are given to 3 boys so that each gets equal
number of things. The probability that the same boy does not receive both the pens is

5 7 2 6
(A) (B) (C) (D)
11 11 3 11
5. Two distinct numbers are selected at random from the first twelve natural numbers.
The probability that the sum will be divisible by 3 is

1 23 1 2
(A) (B) (C) (D)
3 66 2 3
6. The probability of a number n showing in a throw of a dice marked 1 to 6 is proportional
to n. Then the probability of the number 3 showing in a throw is

1 1 1 1
(A) (B) (C) (D)
2 6 7 21
7. The probability that out of 10 persons, all born in April, at least two have the same
birthday is
30 10
C10 C10 30  30  30 C10  30 
10

(A) 10 (B) 1  (C) 10 (D)


 30  30!  30  30!
8. If one ball is drawn at random from each of the three boxes containing 3 white and 1
black, 2 white and 2 black, 1 white and 3 black balls then the probability that 2 white
and 1 black ball will be drawn is
13 1 1 3
(A) (B) (C) (D)
32 4 32 16
9. Four numbers are nultiplied together. Then, the probability that the product will be
divisible by 5 or 10 is

369 399 123 133


(A) (B) (C) (D)
625 625 625 625

Page # 77
PROBABILITY
10. In a multiple choice question there are four alternative answers of which one or
more than one is correct. A candidate will get marks on the question only if he ticks
the correct answer. The candidate decides to tick answers at random. If he is allowed
up to three chances to answer the answer the question, then the probability that he
will get marks on it is
(A) 1/3 (B) 2/3 (C) 1/5 (D) 2/15.
11. The probability that the birth days of six persons will fall in exactly two calendar
months is-

12 26 12 26  1 341
(A) 1/6 (B) C2  (C) C2  (D)
126 126 125

1+ 4p 1- p 1- 2p
12. If , , are probabilities of three mutually exclusive events, then-
4 4 2

1 1 1 2 1 1
(A) p (B) p (C) p (D) none of these
3 2 2 3 6 2
13. A bag contains 50 tickets numbered 1, 2, 3, ..., 50 of which five are drawn at
random and arranged in ascending order of magnitude (x 1 < x2 < x3 < x4 < x5).
The probability that x 3 = 30 is-
20 29 20
C2 C2 C2 29 C2
(A) 50 (B) 50 (C) 50 (D) none of these
C5 C5 C5
14. A man takes a step forward with probability 0.4 and backward with probability
0.6.The probability that at the end of eleven steps he is one step away from the
starting point is -
11
(A) C6 (0.24)5 (B) 11
C6 (0.4)6  0.6 
5

11 5
(C) C6 (0.6)6  0.4  (D) none of these.
15. A biased die is tossed and the respective probabilities for various faces to turn up
are-
Face : 1 2 3 4 5 6
Probability : 0.1 0.24 0.19 0.18 0.15 0.14
If an even face has turned up, then the probability that it is face 2 or face 4, is
(A) 0.25 (B) 0.42 (C) 0.75 (D) 0.9
16. For three events A, B and C, P (exactly one of the events A or B occurs) = P
(exactly one of the events B or C occurs) = P (exactly one of the events C or A
occurs)= p and P (all the three events occur simultaneously) = p 2 ,where 0 < p
< 1/2. Then the probability of atleast one of the three events A,B and C occurring
is-

3p  2p2 p  3p2 3p  p2 3p  2p2


(A) (B) (C) (D)
2 2 2 4
17. If A and B are two events such that P (A) = 1/3, P(B) = 1/4 and P (A  B) = 1/5,
B
then P  

A
(A) 37/40 (B) 37/45 (C) 23/40 (D) None

Page # 78
PROBABILITY
18. There are 9999 tickets bearing numbers 0001, 0002,...., 9999. If one ticket is
selected from these tickets at random, the probability that the number on the
ticket will consist of all different digits, is-
(A) 5040 / 9999 (B) 5000 / 9999 (C) 5030 / 9999 (D) none of these
19. All possible 6 letter words each containing all the letters of the word EMHORT
are formed and are placed in a dictionary order. 10 consecutive words are now
drawn at random. The probability that the word 'MOTHER' will be among the
drawn words, is
(A) 1/72 (B) 10/711 (C) 10/719 (D) none
20. A number is chosen at random from the numbers 10 to 99. By seeing the
number a man will laugh if product of digits is 12. If he chooses three numbers
with replacement then the probability that he will laugh atleast once is-
3 3 3 3
3  43   4   43 
(A) 1    (B)   (C) 1    (D) 1   
5  45   25   45 

MULTIPLE CORRECT TYPE QUESTIONS


1. Cards are drawn one by one without replacement untill two aces are drawn. Let P(m)
be the probability that the event occurs in exactly m trials, then P(m) must be zero at
(A) m = 2 (B) m = 50 (C) m = 51 (D) m = 52
2. A number is chosen at random from the set of integer 1, 2, 3, . . . n. Let A and B be
the events that the number drawn is divisible by 2 and 3 respectively. Then
(A) A and B are always independent (B) A and B are independent if n = 6k
(C) A and B are dependent if n = 10 (D) A and B are independent if n= 6k + 2
3. Let p be the probability the in a pack of playing cards two kings are adjacent and q be
the probability that no two kings are together, then
48  47  46
(A) p = q (B) p < q (C) p + q = 1 (D) q 
52 51 50
4. Which of the following statements are true?
(A) The probability that birthday of twelve people will fall in 12 calender months
12!
=
126
(B) The probability that birthday of six people will fall in exactly two calender months

is 12
C2
2
6
2 
12
6
(C) The probability that birthday of six people will fall is exactly two calender months

is 12
C2
2
6
2 
6 365
12 Pn
(D) The probability that birthday of n (n  365) people are different is
 365 n
5. A bag contains N tickets numbered 1, 2, 3, . . ., N. If r tickets are drawn one by one
with replacement then the probability that all different numbers are drawn is
N
Cr N  N  1 N  2 . . .  N  r  1
(A) N (B)
Pr Nr

Pr 1
(C) (D)
Nr r!
Page # 79
PROBABILITY
6. Which of the following statements are true for two events A and B of the same
sample space?
(A) P  A  B   0 , if A and B are independent

(B) P  A  B   0 , if A and B are mutually exclusive.

(C) P  A  B   P  A   P  B   1

(D) P  A  B   P  B / A 
7. A die is thrown twice. Let X1 and X2 be the outcomes of these trials respectively.
Consider the following events
A1 = {X1 is divisible by 2, X2 is divisible by 3}
A2 = {X1 is divisible by 3, X2 is divisible by 2}
A3 = {X1 is divisible by X2}
A4 = {X2 is divisible by X1}
A5 = {X1 + X2 is divisible by 2}
A6 = {X1 + X2 is divisible by 3}. Then
(A) A1 and A2 are independent (B) A1 and A5 are independent
(C) A3 and A4 are independent (D) A3 and A6 are independent
8. If M and N are any two events, the probablility that exactly one of them occur is
(A) P  M   P  N   2P  M  N  (B) P  M   P  N   P  M  N 

(C) P  M   P  N   P  M  N  (D) P  M  N   P  M  N 

9. Let 0  P  A   1, 0  P  B   1 and P  A  B   P  A   P  B   P  A  P  B  , then

(A) P  B / A   P  B   P  A  (B) P  A   B    P  A    P  B  

(C) P  A   B    P  A   P  B   (D) P  A / B   P  A 
10. Let P(n) be the probability of getting n heads when a coins is tossed m times, if P(4),
P(5), P(6) are in A.P., then the possible values of m could be
(A) 10 (B) 11 (C) 7 (D) 14
11. A five-digit number is written down at random. The probability that the number is
divisible by 5 and no two consecutive digits are identical, is
3 4
1 1  9  3 3
(A) (B) .  (C)   (D) .
5 5  10  5 5
12. Each of the n bags contains a white and b black balls. One ball is transferred from
1st bag to the second bag then one ball is transferred from second bag to the third
bag and so on. Let pn be the probability that ball trnasferred from nth bag is white,
then
a a a a
(A) p1  (B) p2  (C) p3  (D) p4  .
ab ab ab a b
13. In a single cast with two dice the odds against drawing 7 is

1 1
(A) (B) (C) 5 : 1 (D) 1 : 5.
6 12
14. 7 white balls and 3 black balls are placed in a row at random. The probability that no
two black balls are adjacent is
1 7 2 1
(A) (B) (C) (D) .
2 15 15 3
Page # 80
PROBABILITY
15. Given that x   0, 1 and y   0, 1 . Let A be the event of (x, y) satisfying y 2  x and B be
the event of (x, y) satisfying x 2  y . Then

1
(A) P  A  B   (B) A, B are exhaustive
3
(C) A, B are mutually exclusive (D) A, B are independent.
16. 10 apples are distributed at random among 6 persons. The probability that at least
one of them will receive none is
14
6 C4 137 135
(A) (B) 15
C5
(C) (D)
143 143 143
17. 4 gentlemen and 4 ladies take seats at random round a table. The probability that
they are sitting alternately is

4 1 2 1
(A) (B) (C) (D)
35 70 35 35
18. Let x = 33n. The index n is given a positive integral value at random. The probability
that the value of x will have 3 in the units place is
1 1 1 2
(A) (B) (C) (D) .
4 2 3 3
19. Three dice are thrown simultaneously. The probability of getting a sum of 15 is
1 5 5 5
(A) (B) (C) (D) .
72 36 72 108
20. If E and F are the complementary events of the events E and F respectively then

(A) P  E / F   P  E / F   1 (B) P  E / F   P  E / F   1

(C) P  E / F   P  E / F   1 (D) P  E / F   P  E / F   1
21. Three dice are thrown. The probability of getting a sum which is a perfect square is
2 9 1 17
(A) (B) (C) (D)
5 20 4 108
22. The probability of getting a sum of 12 in four throws of an ordinary dice is
3 4 2 3
15 5 1 5 5
(A)   (B)   (C)   (D)  
66 6 36  6  6
23. Three different numbers are selected at random from the set A = {1, 2, 3, …, 10}. The
probability that the product of two of the numbers is equal to the third is

3 1 1 1
(A) (B) (C) (D)
4 40 8 4

3 1 3
24. If A and B are two events such that P  A  B   and  P  A  B  then
4 8 8

11 3
(A) P  A   P  B   (B) P  A   P  B  
8 8

7 1
(C) P  A   P  B   (D) P  A   P  B  
8 8

Page # 81
PROBABILITY
25.. There are 7 seats in a row. Three persons take seats at random. The probability that
the middle seat is always occupied and no two persons are consecutive is
9 9 4 2
(A) (B) (C) (D) .
70 35 34 35
26. A second-order determinant is written down at random using the numbers 1, –1 as
elements. The probability that the value of the determinant is nonzero is
1 3 5 1
(A) (B) (C) (D) .
2 8 8 3
27. If E and F are two events with P(E)  P(F) > 0 then
(A) occurrence of E  occurrence of F
(B) occurrence of F  occurrence of E
(C) nonoccurrence of E  nonoccurrence of F
(D) none of the above implications hold.

28. If two events A and B are such that P  A c   0.3 , P(B) = 0.4 and P  ABc   0.5 , then

P B /  A  BC   is

(A) less than 0.3 (B) 1/4 (C) 1/6 (D) 1/7.
29. Numbers 1, 2, 3, …, 100 are written down on each of the cards A, B and C. One
number is selected at random from each of the cards. The probability that the numbers
so selected can be the measures (in cm) of three sides of right-angled triangles no
two of which are similar, is
2
4 3 3! 1  3 
(A) (B) (C) (D)  
1003 503 1003 100  50 
30. The probabilities that a student passes in mathematics, physics and chemistry are
m, p and c respectively. Of these subjects, a student has a 75% chance of passing in
at least one, a 50% chance of passing in at least two, and a 40% chance of passing in
exactly two subjects. Which of the following relations are true?

19 27 1 1
(A) p  m  c  (B) p  m  c  (C) pmc  (D) pmc 
20 20 10 4

1
31. Let A and B be two events such that P  A  B   1 , P  A  B   5 and P  A   . Then
3 6 2
(A) A, B are independent (B) A, B are mutually exclusive
(C) P(A) = P(B) (D) P  B   P  A 
32. The probability that exactly one of the independent events A and B occurs is equal to
(A) P  A   P  B   2P  A  B  (B) P  A   P  B   P  A  B 

(C) P  A   P  B   2P  A  B  (D) P  A   P  B   P  A  B 

33. If A and B are independent events such that 0 < P(A) < 1, 0 < P(B) < 1 then
(A) A, B are mutually exclusive (B) A and B are independent

(C) A, B are independent (D) P  A / B   P  A / B   1

Page # 82
PROBABILITY
34. For any two events A and B
(A) P  A  B   P  A   P  B   1 (B) P  A  B   P  A   P  B 

(C) P  A  B   P  A   P  B   P  A  B  (D) P  A  B   P  A   P  B   P  A  B 
35. A coin is tossed repeatedly. A and B call alternately for winning a prize of Rs. 30 One
who calls correctly first wins the prize. A starts the cell. Then the expectation of
(A) A is Rs 10 (B) B is Rs 10 (C) A is Rs 20 (D) B is Rs 20.

COMPREHENSION TYPE QUESTIONS


Passage - I:
The probability of happening of an event in one trial being known, then the probability
of its happening exactly x times in n trials is given by nCx qn-x . px where
p = probability of happening the event
q = probability of not happening the event = 1- p.
Now nCx qn-x px is (x + 1)th term in the expansion of (q + p)n whose expansion gives the
happening of the event 0, 1, 2, .... n times respectively.
1. In four throws with a pair of dice, the chance of throwing doublets atleast twice is

19 125 17 18
(A) (B) (C) (D)
144 144 144 144
2. A man takes a forward step with probability (.8) and backward step with probability
(.2). What is the probability that at the end of 9 steps he is exactly three steps away
from starting point

69888 5377 5378 5376


(A) (B) (C) (D)
58 58 58 58
3. Unbiassed coin is tossed 6 times. The probability of getting utmost 4 heads is
7 57 21 11
(A) (B) (C) (D)
64 64 32 32
Passage - II
A commander of an army battalion is punishing two of his soldiers X and Y. He
arranged a duel between them. The rules of the duel are that they are to pick up
their guns and shoot at each other simultaneously.
If one or both hit, then the duel is over. If both shot miss then they repeat the
process. Suppose that the results of the shots are independent and that each shot of
X will hit Y with probability 0.4 and each shot of Y will hit X with probability 0.2. Now
answer the following questions.
4. The probability that the duel ends after first round is
(A) 11/25 (B) 12/25 (C) 13/25 (D) 2/5
5. The probability that X is not hit, is
(A) 3/25 (B) 7/25 (C) 5/13 (D) 8/13
6. The probability that both the soldiers are hit, is
(A) 5/13 (B) 2/13 (C) 8/13 (D) 1/13

Page # 83
PROBABILITY
Passage - III
Let E1, E2, E3 , ….. En be a set of mutually exclusive and exhaustive events and A be

 A
n P(E j ).P  
A E  Ej 
an event, then P(A) =  P(E i ).P   and P  j     for j = 1, 2, … n, where
E
 i
i 1
 A P(A)

F
P   , denotes the probability of occurring the event F given that G has already
G
occurred. There are two bags of red and yellow colours. Red bag contains 4 fair coins
and 3 biased coins and yellow bag contains 5 fair coins and 7 biased coins. Biased
coin has tail on both sides. Two coins are transferred from red bag to yellow bag and
then a coin is taken from yellow bag and tossed.
7. Probability that both coins, transferred from red bag to yellow bag, were fair, is
(A) 1/7 (B) 3/7 (C) 4/7 (D) 2/7
8. Probability that both coins, transferred from red bag to yellow bag, were of mixed
type, is
(A) 3/7 (B) 4/7 (C) 2/7 (D) 1/7
9. If both coins transferred from red bag to yellow bag were biased, then the probability
that tossing of coin results in head, is
(A) 23/28 (B) 9/28 (C) 5/28 (D) 19/28

MATRIX MATCH TYPE QUESTIONS


1. A is a set containing n elements. A subset P of A is chosen at random. The set A is
reconstructed by replacing the elements of the subset P. A subset Q of A is again
chosen at random. The probability that
List - I List-II
(A) P  Q   (p) n(3n – 1)/4n
(B) P  Q is a singleton (q) (3/4)n
2n
(C) P  Q contains 2 elements (r) Cn/4n
(s) 3  n n  1  / 2  4 
n2 n
(D) |P| = |Q|
where |X| = number of elements in X
2. A player tosses a coin and is to score one point for every head and two points for
every tail turned up. He is to play unitl his score reaches or passes n. P n is the
chance of obtaining exactly a score of n, then
Column I Column II
(A) P1 equals to (p) 5/8
(B) P2 equals to (q) 1/2
(C) P3 equals to (r) 3/4
(D) P4 equals to (s) 11/16

Page # 84
PROBABILITY
3. Two dice are thrown. Let A be the event that sum of the points on the two dice is
odd and B be the event that atleast one 3 is there, then match the following
Column I Column II
(A) P(AB) (p) 12/36
(B) P(AB) (q) 6/36
(C) P(A  B) (r) 23/36
(D) P(B) (s) 11/36

INTEGER TYPE QUESTIONS


1. A managing committee consists of 13 experts taken from 3 institutions x, y and z
of which 5 from x, 4 from y and 3 from z. If three experts resign, the probability
k
that they belong to different institutions is , the value of k is ........................
11
2. In a conference, 10 delegates are to be seated around a round table. The probability
that delegates A & B never sit together is K. Then, the value of 9 K – 2 is ____.
3. There are 2n bags each containing 2n balls. ith bag contains i red and 2n- i white
balls. A ball is drawn from one bag and is found to be red. If the probability that it
1
was drawn from nth bag is , then n is equal to____.
7
4. A relation is chosen from all possible relations from A = {1, 2, 3} to ‘A’. If the
probability that the chosen realtion is a function is k3, then the value of 8k + 2 =
.............
5. India & Australia play a series of 7 one day matches. Each team has equal probability
of winning a match. No match ends in a draw. If the probability that India wins
p
atleast 3 consecutive matches can be expressed as
q where p, q are relatively
prime possitive integers. Then the unit digit of p is__________
6. A fair coin is tossed 10 times. If the probability that heads never occur on consecutive
m
tosses be (where m,n are coprime and m, n  N ) then the value of ‘n – 7m’ equals
n
to ....................
7. There are 3 different pairs of shoes in a lot. if 3 persons come and pick the shoes
randomly (each one two shoes). The probability that no one gets correct pair is p.
13p
The value of equals to ...........
4p
8. A fair coin is tossed 12 times. if the probability that two heads do not occur
 4069p  1
consecutively is p, then the value of  
.  denotes G.I.F  is .............
2
9. X and Y are two weak students in Mathematics and their chances of solving a problem
1 1
correctly are & respectively. They are given a questions and they obtain the
8 12

Page # 85
PROBABILITY
1
same answer. If the probability of common mistake is , then the probabiltity
1001
a
that the answer was correct is (a, b being coprimes).then a  b  ........
b
10. A bag contains 2 red, 3 green and 4 black balls. 3 balls are drawn randomly and
exactly two fo them are found to be red. If ‘p’ denotes the chance that one of the
three balls drawn is green, the value of 7p is........

Questions asked in previous AIEEE / JEE MAINS


1. If the probability of solving a problem by three students are 1/2, 2/3 and 1/4 then
probability that the problem will be solved- [AIEEE 2002]
(A) 1/2 (B) 3/4 (C) 7/8 (D) 1/8

2. If P (A  B) = 3/4 and P ( A ) = 2/3 then P ( A  B) equals - [AIEEE 2002]


(A) 1/12 (B) 7/12 (C) 5/12 (D) 1/2
3. A pair of dice is thrown. If 5 appears on at least one of the dice, then the probability that
the sum is 10 or greater, is- [AIEEE 2002]
(A) 11/36 (B) 2/9 (C) 3/11 (D) 1/12
3x  1 1 x
4. Events A, B, C are mutually exclusive events such that P(A) = , P(B) = and
3 4
1  2x
P(C) = . The set of possible values of x are in the interval- [AIEEE 2003]
2

1 1  1 2  1 13 
(A) [0, 1] (B)  ,  (C)  ,  (D)  , 
3 2 3 3 3 3 
5. Five horses are in a race. Mr. A selects two of the horses at random and bets on
them. The probability that Mr. A selected the winning horse is- [AIEEE 2003]
(A) 2/5 (B) 4/5 (C) 3/5 (D) 1/5
6. The probability that A speaks truth is 4/5, while this probability for B is 3/4. The
probability that they contradict each other when asked to speak on a fact is-
[AIEEE 2004]
(A) 3/20 (B) 1/5 (C) 7/20 (D) 4/5
7. A random variable X has the probability distribution : [AIEEE 2004]

For the events E = {X is a prime number} and F= {X < 4}, the probability P(E  F) is-
(A) 0.87 (B) 0.77 (C) 0.35 (D) 0.50
8. The mean and the variance of a binomial distribution are 4 and 2 respectively. Then
the probability of 2 successes is- [AIEEE 2004]
37 219 128 28
(A) (B) (C) (D)
256 256 256 256
9. Three houses are available in a locality. Three persons apply for the houses. Each
applies for one house without consulting others. The probability that alll the three
apply for the same house is - [AIEEE-2005]

Page # 86
PROBABILITY
2 1 8 7
(A) (B) (C) (D)
9 9 9 9
1 1 1
10. Let A and B be two events such that P ( A  B ) = , P (A  B) = and P ( A ) = , where
6 4 4
A stands for complement of event A. Then events A and B are - [AIEEE-2005]
(A) equally likely and mutually exclusive
(B) equally likely but not independent
(C) independent but not equally likely
(D) mutually exclusive and independent
11. A pair of fair dice is thrown independently three times. The probability of getting a
score of exactly 9 twice is- [AIEEE 2007]
(A) 1/729 (B) 8/9 (C) 8/729 (D) 8/243
12. Two aeroplanes I and II bomb a target in succession. The probabilities of I and
II scoring a hit correctly are 0.3 and 0.2, respectively. The second plane will
bomb only if the first misses the target. The probability that the target is hit
by the second plane is- [AIEEE 2007]
(A) 0.06 (B) 0.14 (C) 0.2 (D) 0.7
13. A die is thrown. Let A be the event that the number obtained is greater than 3. Let B
be the event that the number obtained is less than 5. Then P(A  B) is
[AIEEE 2008]
2 3
(A) 0 (B) 1 (C) (D)
5 5
1 1 2
14. It is given that the events A and B are such that P(A) = , P(A|B) = and P(B|A) = .
4 2 3
Then P(B) is [AIEEE 2008]
1 2 1 1
(A) (B) (C) (D)
3 3 2 6

 1
15. In a binomial distribution B  n, p   , if the probability of at least one success is
 4
9
greater than or equal to , then n is greater than : [AIEEE 2009]
10

1 9 4 1
(A) log 4  log 3 (B) log 4  log 3 (C) log 4  log 3 (D) log 4  log 3
10 10 10 10 10 10 10 10

16. One ticket is selected at random from 50 tickets numbered 00, 01, 02, ......., 49. Then
the probability that the sum of the digits on the selected ticket is 8, given that the
product of these digits is zero, equals : [AIEEE 2009]

1 5 1 1
(A) (B) (C) (D)
7 14 50 14
17. Four numbers are chosen at random (without replacement) from the set {1,2,3,.....,20}.
Statement -1 : The probability that the chosen numbers when arranged in some
1
order will form an AP is . [AIEEE 2010]
85
Statement -2 : If the four chosen numbers form an AP, then the set of all possible
values of common difference is {±1, ±2, ±3, ±4, ±5}

Page # 87
PROBABILITY
(A) Statement -1 is true, Statement-2 is true ; Statement -2 is not a correct expla-
nation for Statement -1.
(B) Statement-1 is true, Statement-2 is false.
(C) Statement -1 is false, Statement -2 is true.
(D) Statement -1 is true, Statement -2 is true; Statement-2 is a correct explanation
for Statement-1.
18. An urn contains nine balls of which three are red, four are blue and two are green.
Three balls are drawn at random without replacement from the urn. The probability
that the three balls have different colours is [AIEEE 2010]

2 1 2 1
(A) (B) (C) (D)
7 21 23 3

19. Consider 5 independent Bernoulli’s trials each with probability of success p. If the
31
probability of at least one failure is greater than or equal to , then p lies in the
32
interval [AIEEE 2011]

 3 11   1  11   1 3
(a)  , (b) 0,  (c)  ,1 (d)  , 
 4 12  2   12  2 4
 
20. Three numbers are chosen at random without replacement from {1, 2, 3, …. 8}. The
probability that their minimum is 3, given that their maximum is 6, is
[AIEEE 2012]

3 1 1 2
(a) (b) (c) (d)
8 5 4 5
21. A multiple choice examination has 5 questions. Each question has three alternative
answers of which exactly one is correct. The probability that a student will get 4 or
more correct answers just by guessing is [JEE Mains 2014]

13 11 10 17
(a) (b) (c) (d)
35 35 35 35

22. Let A and B be two events such that P(A  B)  1 / 6, P(A  B)  1 / 4 and P(A)  1 / 4 , where

A stands for the complement of the event A. Then the events A and B are
(a) independent but not equally likely [JEE Mains 2014]
(b) independent and equally likely
(c) mutualy exclusive and independent
(d) equally likely but not independent
23. If 12 identical balls are to be placed in 3 identical boxes, then the probability that one
of the boxes contains exactly 3 balls is : [JEE Mains 2015]
11 10 12 11
55  2   2 1 1
(a)   (b) 55   (c) 220   (d) 22  
3 3  3 3 3

Page # 88
PROBABILITY

Questions asked in previous AIEEE / JEE MAINS


1. Let 0 < P(A) < 1, 0 < P(B) < 1 and P( A  B)  P ( A)  P ( B)  P ( A).P( B) , then [IIT - 95]

(A) P(B/A) = P(B) - P(A) (B) P( A c  Bc )  P ( A c )  P ( Bc )

(C) P(( A  B) c )  P (A c )  P ( Bc ) (D) P(A/B) = P(A)


2. The probability of India wining a test match against West Indies is 1/2. Assuming
independence from match the probability that in a 5 match series India's second win
occurs at the third test is [IIT - 95]
(A) 1/8 (B) 1/4 (C) 1/2 (D) 1/3
3. For the three events A, B and C, P(exactly one of the events A or B occurs) = P(exactly
one of the events B or C occurs) = P(exactly one of the events C or A occurs) = p and
P(all the three events occur simultaneously ) = p 2, where 0 < p < 1/2. Then the
probability of at least one of three events A, B and C occurring is [IIT - 97]
3p  2p 2 p  3p 2 p  3p 2 3p  2p 2
(A) (B) (C) (D)
2 4 2 4

3 1 1
4. If P(B) = , P( A  B  C )  and P( A  B  C )  , then P( B  C) is [IIT - 2003]
4 3 3
(A) 1/12 (B) 1/6 (C) 1/15 (D) 1/9
5. Two number is selected randomly from the set S = {1, 2, 3, 4, 5, 6}without replace-
ment one by one. The probability that minimum of the two numbers is less than 4 is
[IIT - 2003]
1 14 1 4
(A) (B) (C) (D)
15 15 5 5
6. If three distinct numbers are chosen randomly from the first 100 natural numbers,
then the probability that all three are divisible by both 2 and 3 is [IIT - 2004]
(A) 4/25 (B) 4/35 (C) 4/33 (D) 4/1155
7. A six faced fair die is thrown until 1 comes, then the probability that 1 comes in even
number of trials is [IIT - 2005]
(A) 5/11 (B) 5/6 (C) 6/11 (D) 1/6
8. An experiment has 10 equally likely outcomes. Let A and B be two non-empty events
of the experiment. If A consists of 4 outcomes, the number of outcomes that B must
have so that A and B are independent, is
(A) 2, 4 or 8 (B) 3, 6 or 9 (C) 4 or 8 (D) 5 or 10 [IIT 2008]
9. Let w be a complex cube root of unity with w ¹ 1. A fair die is thrown three times. If
r1, r2 and r3 are the numbers obtained on the die, then the probability that r  r  r = 0 1 2 3

is [IIT 2010]
1 1 2 1
(A) (B) (C) (D)
18 9 9 36

4 1
10. A signal which can be green or red with probability and respectively, is received
5 5
by station A and then transmitted to station B. The probability of each station receiv-
3
ing the signal correctly is . If the signal received at station B is green, then the
4
probability that the original signal was green is [IIT 2010]

Page # 89
PROBABILITY
3 6 20 9
(A) (B) (C) (D)
5 7 23 20
11. Four fair dice D1, D2, D3 and D4, each having six faces numbered 1, 2, 3, 4, 5, and 6,
are rolled simultaneously. The probability that D4 shows a number appearing on one
of D1, D2 and D3 is [IIT 2012]
91 108 125 127
(A) (B) (C) (D)
216 216 216 216
12. Four persons independently solve a certain problem correctly with probabilities
1 3 1 1
, , , . Then the probability that the problem is solved correctly by at least one of
2 4 4 8
them is [JEE Advanced 2013]
235 21 3 253
(A) (B) (C) (D)
256 256 256 256

SUBJECTIVE TYPE QUESTIONS


13. In a test an examinee either guesses or copies or knows the answer to a multiple
1
choice question with four choices. The probability that he make a guess is and the
3
1
probability that he copies the answer is . The probability that his answer is correct
6
1
given that he copied it, is . Find the probability that he knew the answer to the
8
question given that he correctly answered it. [IIT - 91]
14. Numbers are selected at random, one at a time, from the two digit numbers
00, 01, 02, ............ ,99 with replacement. An event E occurs if and only if the product
of the two digits of a selected number is 18. If four numbers are selected, find the
probability that the event E occurs at least 3 times. [IIT - 93]
15. An unbiased coin is tossed. If the result is a head, a pair of unbiased dice is rolled
and the number obtained by adding the numbers on the two faces is noted. If the
result is a tail, a card from a well shuffled pack of eleven cards numbered 2, 3,
4,.........,12 is picked and the number on the card is noted. What is the probability
that the noted number is either 7 or 8. [IIT - 94]
16. In how many ways 3 girls and 9 boys can be seated in two vans, each having num-
bered seats, 3 in the front and 4 at the back ? How many seating arrangements are
possible if 3 girls should sit together in a back row on adjacent seats ? Now, if all the
seating arrangements are equally likely, what is the probability of 3 girls sitting
together in a back row on adjacent seats ? [IIT-96]
17. 3 players A, B and C toss a coin cyclically in that order (that is A, B, C, A, B, C, A,
B,........) till a head shows. Let p be the probability that the coin shows a head. Let
,  and  be respectively the probabilities that A, B and C gets the first head. Prove
that   (1  p)  . Determine ,  and  (in terms of p). [IIT - 98]
18. A coin has probability 'p' of showing head when tossed. It is tossed 'n' times. Let P n
denote the probability that no two (or more) consecutive heads occur. Prove that
p1 = 1, p2 = 1 - p2 and pn = (1 - p)pn - 1 + p(1 - p)pn - 2 , for all n  3 . [IIT - 2000]

Page # 90
PROBABILITY
19. An urn contains 'm' white and 'n' black balls. A ball is drawn at random and is put
back into the urn along with K additional balls of the same colour as that of the ball
drawn. A ball is again drawn at random. What is the probability that the ball drawn
now is white? [IIT - 2001]
20. An unbiased die, with faces numbered 1, 2, 3, 4, 5, 6 is thrown n times and the list of
n numbers showing up is noted. What is the probability that among the numbers 1, 2,
3, 4, 5, 6 only three numbers appear in the list. [IIT - 2001]
21. A box contains N coins, m of which are fair and the rest are biased. The probability of
getting a head when a fair coin is tossed is 1/2, while it is 2/3 when a biased coin is
tossed. A coin is drawn from the box at random and is tossed twice. The first time it
shows head and the second time it shows tail. What is the probability that the coin
drawn is fair ? [IIT - 2002]
22. For a student to qualify, he must pass at least two out of three exams. The probability
that he will pass the 1st exam is p. It he fails in one of the exams then the probability
p
of his passing in the next exam is otherwise it remains the same. Find the prob-
2
ability that he will qualify. [IIT - 2003]
23. A is targeting to B, B and C are targeting to A. Probability of hitting the target by A, B
2 1 1
and C are , and respectively. If A is hit then find the probability that B hits the
3 2 3
target and C does not. [IIT - 2003]
24. There are 18 balls, 12 red and 6 white. Six balls are drawn one by one without
replacement. If at least 4 are white, find the probability that next two draw will result
in one red and one white ball. [IIT - 2004]
25. A person goes to office either by car, scooter, bus or train, the probability of which
1 3 2 1
being , , and respectively. Probability that he reach office late, if he takes car,
7 7 7 7
2 1 4 1
scooter, bus or train is , , and respectively. Given that he reached office in
9 9 9 9
time, then what is the probability that he travelled by a car? [IIT - 2005]
26. Of the three independent events E1, E2, and E3, the probability that only E1 occurs is
 only E2 occurs is  and only E3 occurs is  . Let the probability p that none of events
E1, E2 or E3 occurs satisfy the equations (  2)p   and (  3 )p  2 . All the given
probabilities are assumed to lie in the interval (0, 1).
Probability of occurrence of E1
Then Probability of occurrence of E  [JEE Advanced - 2013]
3

27. The minimum number of times a fair coin needs to be tossed, so that the probability
of getting at least two heads is at least 0.96, is. [JEE Advanced - 2015]

COMPREHENSIVE TYPE QUESTIONS


I. There are n urns numbered1, 2,, ....., n each containing (n + 1) balls. Urn i contains
i white balls and (n + 1 – i) red balls, i = 1,2, .....n. An urn is selected and a ball is
drawn at random from it. Let Ui denote the event that urn numbered i is selected and
let W denote the event that a white ball is drawn from the selected urn. Further,
suppose that E denotes the event that an even numbered urn is selected [IIT 2006]
28. If P(Ui)  i, i = 1,2,....., n, then lim P(W) equals
n 

2 1 1
(A) (B) (C) (D) 1
3 2 3
Page # 91
PROBABILITY
29. If P(Ui) = c, i = 1, 2,.....,n, where c is a constant, then P(Un/W) equals
i 2 n
(A) (B) (C) (D) 1
n 1 n 1 n 1
30. If n is even and E denotes the event of choosing even numbered urn
 1 
 P  U i   , i  1, 2,...., n  then the value of P  W / E  is
 n 

n2 n2 n2 1


(A) (B) 2 n  1 (C) 2 n  1 (D)
2n  1     n 1
II. A fair die is tossed repeatedly until a six is obtained. Let X denote the number of
tosses required.
31. The probability that X = 3 equals [IIT 2009]

25 25 5 125
(A) (B) (C) (D)
216 36 36 216
32. The probability that X  3 equals

125 25 5 25
(A) (B) (C) (D)
216 36 36 216
33. The conditional probability that X  6 given X > 3 equals

125 25 5 25
(A) (B) (C) (D)
216 216 36 36
III. Let U1 and U2 be two urns such that U 1 contains 3 white and 2 red balls, and U 2
contains only 1 white ball. A fair coin is tossed. If head appears then 1 ball is drawn
at random from U1 and put into U2. However, if tail appears then 2 balls are drawn at
random from U1 and put into U2. Now 1 ball is drawn at random from U2. [IIT 2011]
34. The probability of the drawn ball from U2 being white is
(A) 13/30 (B) 23/30 (C) 19/30 (D) 11/30
35. Given that the drawn ball from U2 is white, the probability that head appeared on
the coin is
(A) 17/23 (B) 11/23 (C) 15/23 (D) 12/23
IV. A box B1 contains 1 white ball, 3 red balls and 2 black balls. Another box B2 contains
2 white balls, 3 red balls and 4 black balls. A third box B3 contains 3 white balls, 4 red
balls and 5 black balls. [JEE Advanced 2013]
36. If 1 ball is drawn from each of the boxes B1, B2 and B3, the probability that all 3
drawn balls are of the same colour is

82 90 558 566
(A) (B) (C) (D)
648 648 648 648
37. If 2 balls are drawn (without replacement) from a randomly selected box and one of
the balls is white and the other ball is red, the probability that these 2 balls are
drawn from box B2 is

116 126 65 55
(A) (B) (C) (D)
181 181 181 181

Page # 92
PROBABILITY
V. Let n1 and n2 be the number of red and black balls, respectively, in box I. Let n3 and n4
be the number of red and black balls, respectively in box II. [JEE Advanced -2015]
38. One of the two boxes, box I and box II, was selected at random and a ball was drawn
randomly out of this box. The ball was found to be red. If the probability that this red
ball was drawn from the box II is 1/3, then the correct option(s) with the possible
values of n1, n2, n3 and n4 is (are)
(A) n1 = 3, n2 = 3, n3 = 5, n4 = 5
(B) n1 = 3, n2 = 6, n3 = 10, n4 = 50
(C) n1 = 8, n2 = 6, n3 = 5, n4 = 20
(D) n1 = 6, n2 = 12, n3 = 5, n4 = 20
39. A ball is drawn at random from box I and transferred to box II. If the probability of
drawing a red ball from box I, after this transfer, is 1/3, then the correct option(s)
with the possible values of n1 and n2 is(are) :
(A) n1 = 4 and n2 = 6 (B) n1 = 2 and n2 = 3
(C) n1 = 10 and n2 = 20 (D) n1 = 3 and n2 = 6

Page # 93
PROBABILITY

ANSWER KEY
DRILL - I
1. B 2. B 3. B 4. C 5. D 6. C 7. B 8. A
9. D 10. C 11. A 12. B 13. D 14. A 15. B 16. B
17. A 18. B 19. B 20. A 21. B 22. C 23. D 24. A
25. A 26. C
DRILL - II
1. C 2. B 3. B 4. D 5. C 6. A 7. D 8. C
DRILL -III
1. B 2. B 3. C 4. A 5. C 6. A 7. C 8. A
9. D 10. B
DRILL - IV
1. A 2. C 3. A 4. D 5. D 6. D 7. D 8. A

DRILL - V
1. B 2. A 3. A 4. C 5. C 6. C 7. B 8. B
9. A 10. B 11. D 12. C 13. B 14. A 15. A 16. A

17. B 18. A 19. B 20. D 21. B 22. B 23. C 24. B


25. A 26. B 27. C 28. D
DRILL - VI

1. D 2. A
DRILL - VII
1. D 2. A 3. D 4. B 5. C 6. D

DRILL - VIII
1. A 2. D 3. D 4. B 5. B 6. A 7. B 8. B
9. B 10. A 11. C 12. A 13. A 14. B 15. A 16. A

17. C
DRILL - IX
1. B 2. D 3. B

SINGLE CORRECT TYPE QUESTIONS


1. D 2. B 3. B 4. A 5. A 6. C 7. C 8. A
9. A 10. C 11. D 12. D 13. C 14. A 15. C 16. A
17. A 18. A 19. B 20. D 21. D 22. A 23. B
MULTIPLE CORRECT TYPE QUESTIONS
1. C, D 2. B, C, D 3. B, D 4. A, C, D 5. B, C 6. B, D 7. A
8. A, D 9. C, D 10. C, D 11. C 12. A, B, C, D 13. C 14. B

. Page # 94
PROBABILITY
15. A 16. C 17. D 18. A 19. D 20. A, D 21. D 22. A
23. B 24. A, C 25. C 26. A 27. D 28. A, B 29. D 30. B, C
31. A 32. A, C 33. B, C, D 34. A, C 35. B, C

COMPREHENSION TYPE QUESTIONS


1. A 2. A 3. B 4. C 5. D 6. B 7. D 8. B
9. C
MATRIX MATCH TYPE QUESTIONS
1. (A – q), (B – p), (C – s), (D – r) 2. (A – q), (B – r), (C – p), (D – s)
3. (A – r), (B – q), (C – p), (D – s)
INTEGER TYPE QUESTIONS
1. 3 2. 5 3. 3 4. 5 5. 7 6. 1 7. 2
8. 9 9. 1 10. 3

PREVIOUS YEAR QUESTIONS (AIEEE)


1. C 2. C 3. C 4. B 5. A 6. C 7. B 8. D
9. B 10. C 11. D 12. B 13. B 14. A 15. D 16. D
17. B 18. A 19. B 20. B 21. B 22. A 23. A

PREVIOUS YEAR QUESTIONS (IIT-JEE)


1. D 2. B 3. A 4. A 5. D 6. D 7. A 8. D

24 97 193
9. C 10. C 11. A 12. A 13. 14. 4 15. = 0.2436
29  25  792
16. (i) 7(13!); (ii) 12!; (iii) 12!/7.13! = 1/91
2


p
,
1  p  p ,   1  p  p
17. 3 3 3
1  1  p  1  1  p  1  1  p 

m 6C3 3n  3  2n   3 9m


18. 24/29 19. 20. 21.
mn 6 n m  8N
1
22. 2p 2  p3 23.
2
 10 C1  2 C1 12
C2  6 C4 11 C1  1C1 12 C1  6C5  1
24.    12  18  = 12

12
C2 18
C6 C2 C6  C2 6 C4 12 C1 6 C5 12 C0 6 C6
25. 7/49 26. (6) 27. (8) 28. A 29. B 30. B 31. A
32. B 33. D 34. B 35. D 36. A 37. D 38. A,B
39. C,D 40. B 41. AD 42. BD

Page # 95

You might also like